Научная статья на тему '2012. 03. 003. Липсет С. М. Некоторые социальные предпосылки демократии: экономическое развитие и политическая легитимность. (перевод). Lipset S. M. Some social requisites of democracy: economic development and political legitimacy // American political science rev. - Cambridge, 1959. - Vol. 53, n 1. - p. 69-105'

2012. 03. 003. Липсет С. М. Некоторые социальные предпосылки демократии: экономическое развитие и политическая легитимность. (перевод). Lipset S. M. Some social requisites of democracy: economic development and political legitimacy // American political science rev. - Cambridge, 1959. - Vol. 53, n 1. - p. 69-105 Текст научной статьи по специальности «Политологические науки»

CC BY
1608
255
i Надоели баннеры? Вы всегда можете отключить рекламу.
Ключевые слова
БЮРОКРАТИЯ / ГРАМОТНОСТЬ / ДЕМОКРАТИЗАЦИЯ ОБЩЕСТВА / ИНДУСТРИАЛИЗАЦИЯ / ЛЕГИТИМНОСТЬ / ЛИПСЕТ С.М. / МОДЕРНИЗАЦИЯ ОБЩЕСТВА / ОБРАЗОВАНИЕ И ОБЩЕСТВО / ОЛИГАРХИЯ / ПОЛИТИЧЕСКАЯ КУЛЬТУРА / ПОЛИТИЧЕСКАЯ СИСТЕМА / РЕЛИГИЯ И ПОЛИТИКА / СОЦИАЛЬНО-ЭКОНОМИЧЕСКОЕ РАЗВИТИЕ / СОЦИОЛОГИЯ ПОЛИТИКИ / СРЕДНИЙ КЛАСС / ТОЛЕРАНТНОСТЬ / УРБАНИЗАЦИЯ / ЭГАЛИТАРИЗМ / ЭКОНОМИКА И ПОЛИТИКА
i Надоели баннеры? Вы всегда можете отключить рекламу.
iНе можете найти то, что вам нужно? Попробуйте сервис подбора литературы.
i Надоели баннеры? Вы всегда можете отключить рекламу.

Текст научной работы на тему «2012. 03. 003. Липсет С. М. Некоторые социальные предпосылки демократии: экономическое развитие и политическая легитимность. (перевод). Lipset S. M. Some social requisites of democracy: economic development and political legitimacy // American political science rev. - Cambridge, 1959. - Vol. 53, n 1. - p. 69-105»

и общества в целом, природы «американской исключительности» и таких характеристик американской культуры, как либеральные и эгалитаристские ценности. Проделанный Липсетом исследовательский путь - от изучения социальных предпосылок демократии, труда и политики до осмысления сути американской культуры и общества - сделал его одним из самых известных политических социологов XX в., заключает Арчер.

М.Е. Соколова

2012.03.003. ЛИПСЕТ СМ. НЕКОТОРЫЕ СОЦИАЛЬНЫЕ ПРЕДПОСЫЛКИ ДЕМОКРАТИИ: ЭКОНОМИЧЕСКОЕ РАЗВИТИЕ И ПОЛИТИЧЕСКАЯ ЛЕГИТИМНОСТЬ1. (Перевод2). LIPSET S.M. Some social requisites of democracy: Economic development and political legitimacy // American political science rev. -Cambridge, 1959. - Vol. 53, N 1. - P. 69-105.

Условия существования и стабильности демократического общества стали главным предметом интереса в политической философии. В настоящей статье эта проблема будет рассмотрена с социологической и поведенческой точки зрения; я выдвину ряд гипотез относительно социальных предпосылок демократии и обсужу некоторые имеющиеся данные, чтобы проверить эти гипотезы. Концентрируясь на условиях - ценностях, социальных институтах, исторических событиях, - внешних для политической системы как таковой и поддерживающих разные общие типы политических систем, эта статья выходит за рамки общепризнанной сферы политической социологии. В этой растущей области усилия были направлены по большей части на внутренний анализ организаций, преследующих политические цели, или на выяснение детерминант действия внутри разных политических институтов, таких как пар-

1 Эта статья была написана как часть сравнительного анализа политического поведения в западных демократиях, при финансовой поддержке отдела поведенческих наук Фонда Форда и Комиссии по сравнительной политике Совета социально-научных исследований. Хочу поблагодарить за помощь Роберта Алфорда и Амитая Этциони. Первоначально статья была представлена как доклад на конференции Американской ассоциации политической науки в сентябре 1958 г. в Сент-Луисе, шт. Миссури.

2 Перевод выполнен с незначительными сокращениями, сделанными в основном за счет таблиц и некоторых примечаний. - Прим. пер.

тии, государственные службы или электоральный процесс1. Более широкий взгляд на связи политической системы с обществом в целом оставался главным образом делом политического философа.

I. Введение

Социологический анализ любого паттерна поведения, относится ли он к малой или крупной социальной системе, должен приводить к конкретным гипотезам, т.е. эмпирически проверяемым утверждениям. Так, при работе с демократией надо суметь указать набор условий, которые действительно существовали в некоторых странах, и сказать: демократия возникала из этих условий и стабилизировалась благодаря ряду поддерживающих институтов и ценностей, а также благодаря своим внутренним процессам самосохранения. Включаемые в список условия должны отличать наиболее демократические государства от большинства других.

Недавняя дискуссия группы политических теоретиков о «культурных предпосылках успешно функционирующей демократии» показывает разницу между подходом политического социол«^-га и политического философа к этой компаративной проблеме . Значительная часть этого симпозиума посвящена дебатам о вкладе религии, особенно христианской этики, в демократические установки. Основной докладчик, Эрнест Гриффит, видит необходимую связь между иудео-христианским наследием и установками, служащими опорой демократических институтов; другие участники выводят на передний план политические и экономические условия, которые могут служить основой для консенсуса по базовым ценностям, не зависящего от религии; они указывают на депрессию, бедность и социальную дезорганизацию, которые привели к фашизму

1 См. мою статью: Lipset S.M. Political sociology, 1945-1955 // Sociology in the USA / Ed. by H.L. Zetterberg. - P.: UNESCO, 1956. - P. 45-55, где дан обзор различных областей, охватываемых политической социологией. Обсуждение интеллектуальных тенденций в политической социологии и оснований для фокусировки на проблеме демократии см. в моей статье: Political sociology // Sociology today / Ed. by R.K. Merton. - N.Y.: Basic books, 1959. - Ch. 3.

Griffith E.S., Plamenatz J., Pennock J.R. Cultural prerequisites to a successfully functioning democracy: A symposium // American political science rev. - Cambridge, 1956. - Vol. 50, N 1. - P. 101-137.

в Италии и Германии, несмотря на очень религиозное население и традиции. Что больше всего поражает в этой дискуссии, так это отсутствие перспективы, полагающей, что теоретические положения должны подлежать проверке с помощью систематического сравнения всех имеющихся случаев, и должным образом трактующей отклоняющийся случай как один из многих. Напротив, в этом симпозиуме отклоняющиеся случаи, не укладывающиеся в то или иное положение, приводятся для демонстрации того, что нет никаких социальных условий, которые были бы регулярно связаны с данной сложной политической системой. В результате конфликты между политическими философами по поводу необходимых условий, лежащих в основе данных политических систем, часто ведут к триумфальной демонстрации того, что та или иная ситуация явно противоречит тезису оппонента, во многом так же, как если бы существование богатых социалистов или бедных консерваторов доказывало, что экономические факторы не являются важной детерми-нантой политических предпочтений.

Преимущество предлагаемой попытки разбить условия демократии на несколько взаимосвязанных переменных состоит в том, что она позволяет правильно толковать отклоняющиеся случаи. Статистическое преобладание данных, подтверждающих связь такой переменной, как образование, с демократией, показывает, что существование отклоняющихся случаев (таких как Германия, скатившаяся к диктатуре, несмотря на передовую систему образования) не может быть достаточной основой для отвержения гипотезы. Отклоняющийся случай, взятый в контексте, включающем данные обо всех релевантных случаях, часто может реально усилить базовую гипотезу, если его интенсивное изучение раскроет особые условия, помешавшие появиться обычной связи1. Так, электоральные

1 Подробно описанный пример того, как отклоняющийся случай и его анализ продвигают вперед теорию, можно найти в книге: Lipset S.M., Trow M., Coleman J. Union democracy. - Glencoe (IL): Free press, 1956. В этой книге исследуется политический процесс внутри Международного союза типографских работников, где давно сложилась двухпартийная система со свободными выборами и частой сменой руководства; таким образом, это явное исключение из «железного закона олигархии» Роберта Михельса. Между тем целью этого исследования был не отчет об этом профсоюзе, а скорее использование его как лучшего из имеющихся средств для проверки и усовершенствования «закона» Михельса. Провести такое исследование можно было лишь при систематической попытке установить базо-

исследования показывают, что немалую часть экономически более состоятельных леваков составляют люди, которые ущемлены в правах; значимы также другие измерения социального статуса, такие как этническая или религиозная позиция.

Споры в этой области вытекают не только из различий в методологии, но и из использования разных определений. Чтобы обсуждать демократию или любой другой феномен, нужно сначала его определить. Для целей этой статьи демократия (в сложном обществе) определяется как политическая система, предоставляющая регулярные конституционные возможности для смены государственных чиновников. Это социальный механизм разрешения проблемы принятия общественных решений среди конфликтующих групп интересов, позволяющий как можно большей части населения влиять на эти решения благодаря возможности делать выбор между альтернативными претендентами на политические посты. Это определение, извлеченное в значительной части из работ Йо-зефа Шумпетера и Макса Вебера1, предполагает несколько специфических условий:

а) «политическую формулу», т.е. систему представлений, легитимирующих демократическую систему и специфицирующих институты - партии, свободную прессу и т.д., - которые легитимируются, т.е. принимаются всеми как должное;

б) один набор политических лидеров, занимающих посты;

в) несколько наборов лидеров, не занимающих никаких постов, которые выступают как легитимная оппозиция, пытающаяся занять эти посты.

Необходимость этих условий очевидна. Во-первых, если политическая система не характеризуется ценностной системой, допускающей мирную «игру» во власть - подчинение оппозиции решениям, принимаемым теми, кто стоит у власти, и признание теми,

вую теорию и вывести гипотезы. Лучшим способом развить знание о внутреннем управлении в добровольных ассоциациях представлялось изучение наиболее де-виантного случая. В процессе выяснения особых исторических и структурных условий, поддерживающих двухпартийную систему в МСТР, была прояснена общая теория.

1 Schumpeter J. Capitalism, socialism and democracy. - N.Y.: Harper & bros., 1947. - P. 232-302, особенно с. 269; Weber M. Essays in sociology. - N.Y.: Oxford univ. press, 1946. - P. 226.

кто стоит у власти, прав оппозиции, - то никакой стабильной демократии быть не может. С этой проблемой столкнулись многие латиноамериканские государства. Во-вторых, если результатом политической игры не является периодическое вознаграждение властными полномочиями одной группы, партии или стабильной коалиции, то следствием этого будет нестабильное и безответственное правительство, а не демократия. Это положение дел существовало в дофашистской Италии и на протяжении значительной части - хотя и не всей - истории Третьей и Четвертой республик во Франции, для которых были характерны слабые коалиционные правительства, формировавшиеся зачастую партиями, между которыми были серьезные конфликты интересов и ценностные конфликты. В-третьих, если не существует условий, способствующих воспроизведению действенной оппозиции, то властные полномочия (authority) чиновничества будут максимизироваться, а влияние народа на проводимую политику будет сведено к минимуму. Такова ситуация во всех однопартийных государствах; и, по общему согласию - по крайней мере на Западе, - это диктатуры.

Здесь будут рассмотрены две основные комплексные характеристики социальных систем, значимые для проблемы стабильной демократии: экономическое развитие и легитимность. Они будут представлены как структурные характеристики общества, поддерживающие демократическую политическую систему. После обсуждения комплекса экономического развития (включающего индустриализацию, благосостояние, урбанизацию и образование) и его последствий для демократии мы перейдем к двум аспектам проблемы легитимности, или того, в какой степени институты ценятся сами по себе и считаются правильными и подобающими. Связи между легитимностью и эффективностью системы (последняя -прежде всего функция экономического развития) будут прослежены путем обсуждения источников раскола в обществе и того, каким образом различные варианты решения критических проблем приводят либо к разрушительным формам раскола, либо к перекрещивающимся аффилиациям, сводящим конфликт до управляемого уровня. И, наконец, будет оценена значимость этих различных факторов для будущего демократии.

Подробного исследования политической истории отдельных стран мы, в соответствии с общим определением, предпринимать

не будем, поскольку относительная степень или социальное содержание демократии не являются темой этой статьи. Между тем некоторые методологические проблемы обращения со связями между комплексными характеристиками целостных обществ заслуживают краткого обсуждения.

Чрезвычайно высокая корреляция между аспектами социальной структуры, такими как доход, образование, религия, с одной стороны, и демократией - с другой, не предвидится даже на теоретических основаниях, ведь политическая подсистема общества функционирует автономно, и коли это так, то конкретная политическая форма может сохраняться при условиях, обычно противоречащих появлению этой формы. Или политическая форма может развиваться вследствие синдрома уникальных исторических факторов, даже если основные социальные характеристики благоприятствуют другой форме. Германия - пример нации, в которой все структурные изменения (растущие индустриализация, урбанизация, благосостояние и образование) благоприятствовали установлению демократической системы, однако ряд неблагоприятных исторических событий помешал демократии заручиться легитимностью в глазах многих важных сегментов общества и тем самым ослабил способность германской демократии противостоять кризису.

Высокие корреляции, проявляющиеся в представленных данных между демократией и другими институциональными характеристиками обществ, не следует переоценивать, поскольку уникальные события могут объяснять как сохранение, так и провал демократии в тех или иных конкретных обществах. Макс Вебер настойчиво утверждал, что различия в национальных паттернах часто являются отражением ключевых исторических событий, приводящих в движение в одной стране один процесс, а в другой -другой. Для пояснения своей позиции он уподоблял это игре в кости, в которой с каждым броском, в котором выпадает какое-то число, центр тяжести в этих кубиках все больше смещается так, чтобы при следующих бросках снова выпадало это число1. Для Вебера событие, предрасполагающее страну к демократии, запускает процесс, повышающий вероятность того, что в следующий критиче-

1 Weber M. The methodology of the social sciences. - Glencoe (IL): Free press, 1949. - P. 182-185. См. также: Lipset S.M. A sociologist looks at history // Pacific sociological rev. - N.Y., 1958. - Vol. 1, N 1. - P. 13-17.

ский момент в истории страны демократия снова одержит верх. Этот процесс может иметь значимость лишь при условии, что, однажды установившись, демократическая политическая система приобретает некоторую инерцию и создает определенные социальные подпорки (институты), гарантирующие ее дальнейшее существование. Таким образом, «преждевременная» демократия выживает (помимо прочего) благодаря тому, что способствует развитию других благоприятствующих ей условий, таких как всеобщая грамотность и автономные частные ассоциации. Главная задача этой статьи - прояснить социальные условия, служащие опорой для демократической политической системы, такие как образование или легитимность; виды внутренних механизмов, служащих поддержанию демократических систем, такие как специфические правила политической игры, в ней подробно рассматриваться не будут1.

Сравнительные обобщения, касающиеся сложных социальных систем, неизбежно обходятся весьма упрощенно с частными историческими чертами любого общества, попадающего в сферу исследования. Для проверки этих обобщений, относящихся к различиям между странами, располагающимися выше или ниже в шкале обладания атрибутами, связанными с демократией, нужно установить какие-то эмпирические параметры для типа политической системы. Отдельные отклонения от того или иного аспекта демократии не так уж важны, пока определения недвусмысленно покрывают большинство наций, опознаваемых как демократические или недемократические. Точная граница между «более демократичными» и «менее демократичными» тоже не является главной проблемой, поскольку предполагается, что демократия - не качество социальной системы, либо существующее, либо не существующее, а скорее комплекс характеристик, которые можно ранжировать множеством разных способов. Поэтому было решено

1 См. новейшие систематические попытки уточнить некоторые из внутренних механизмов демократии: Janowitz M., Marvick D. Competitive pressure and democratic consent: An interpretation of the 1952 presidential election. - Ann Arbor: Univ. of Michigan press, 1956. - (Michigan governmental studies); Dahl R.A. A preface to democratic theory. - Chicago (IL): Univ. of Chicago press, 1956 (особенно с. 90123). Обсуждение проблем внутреннего анализа политических систем см. в: Easton D. An approach to the analysis of political systems // World politics. - Cambridge, 1957. - Vol. 9, N 3. - P. 383-400.

разделить рассматриваемые страны на две группы, вместо того чтобы пытаться проранжировать их по принципу «выше - ниже». Проранжировать отдельные страны от наиболее до наименее демократических гораздо труднее, чем разбить их на два класса, «более» и «менее» демократические, хотя даже здесь пограничные случаи, такие как Мексика, создают проблемы.

Попытки классифицировать все страны порождают ряд затруднений. Большинство стран, в которых отсутствуют прочные традиции политической демократии, находятся в традиционно недоразвитых частях мира. Возможно, Макс Вебер был прав, когда предположил, что современная демократия в ее наиболее чистых формах может проявиться лишь в уникальных условиях капиталистической индустриализации1. Некоторые из усложнений, вносимых колоссальными вариациями политических практик в разных частях земного шара, могут быть устранены, если работать с различиями между странами в пределах политических культурных ареалов. Двумя лучшими ареалами для такого внутреннего сравнения являются, во-первых, Латинская Америка и, во-вторых, Европа и англоязычные страны. Более ограниченные сравнения можно проводить между азиатскими государствами и среди арабских стран.

Основные критерии, используемые в этой статье для распознавания европейских демократий, - это непрерывное воспроизводство политической демократии после Первой мировой войны и отсутствие за последние 25 лет серьезных политических движений, выступающих против демократических «правил игры»2. К Латинской Америке применяется немного менее строгий критерий, состоящий в том, имела ли данная страна историю более или менее свободных выборов на протяжении большей части периода после Первой мировой войны. В то время как в Европе мы ищем стабильные демократии, в Южной Америке мы ищем страны, в которых не было почти постоянного диктаторского правления (см.

1 См.: Weber M. Zur Lage der bürgerlichen Demokratie in Russland // Archiv für Sozialwissenschaft u. Sozialpolitik. - Tübingen, 1906. - Bd. 22. - P. 346 ff.

Последнее требование означает, что ни одно тоталитарное движение - ни фашистское, ни коммунистическое - не получало в это время 20% голосов. На самом деле во всех европейских нациях, находящихся на демократической стороне континуума, были свои тоталитарные движения, но они располагали менее 7% голосов.

табл. 1). Ни в случае Европы, ни в случае Латинской Америки подробного анализа политической истории, который бы учитывал более специфические критерии дифференциации, не проводилось; на этом этапе исследования предпосылок демократии для классификации европейских стран будет достаточно данных о результатах выборов, а для Латинской Америки - суждений экспертов и впечатлений, основанных на хорошо известных фактах политической истории.

Таблица 1

Классификация европейских, англоязычных и латиноамериканских наций по степени демократии

Европейские и англоязычные нации

Латиноамериканские нации

Стабильные демократии

Нестабильные демократии и диктатуры

Демократии и нестабильные диктатуры

Стабильные диктатуры

Австралия Бельгия Великобритания Дания Ирландия Канада Люксембург Нидерланды Новая Зеландия Норвегия

США Швейцария Швеция

Австрия Болгария Венгрия Германия (Западная)

Греция Исландия Испания Италия Польша Португалия Румыния Финляндия Франция Чехословакия Югославия

Аргентина Бразилия Колумбия Коста-Рика Мексика Уругвай Чили

Боливия Венесуэла

Гаити Гватемала Гондурас Доминиканская Республика

Куба Никарагуа Панама Парагвай

Перу Сальвадор Эквадор

II. Экономическое развитие и демократия

Наиболее распространенным обобщением, связывающим демократию с другими аспектами общества, является, пожалуй, то, в котором демократия связывается с состоянием экономического развития. Конкретно это означает, что чем выше благосостояние

нации, тем больше шансов на то, что она будет поддерживать демократию. От Аристотеля до наших дней утверждалось, что только в богатом обществе, где лишь сравнительно немногие граждане живут в настоящей бедности, возможна ситуация, когда широкие массы населения разумно участвуют в политике и развивают самоограничение, необходимое для того, чтобы не поддаваться призывам безответственных демагогов. Итогом разделения общества на широкие обездоленные массы и небольшую привилегированную элиту должна быть либо олигархия (диктаторское правление небольшого верхнего слоя), либо тирания (опирающаяся на народ диктатура). Этим двум политическим формам можно присвоить современные названия-ярлыки: современным лицом тирании являются коммунизм или перонизм, а олигархия проявляет себя сегодня в форме традиционалистских диктатур, таких, какие мы находим в разных частях Латинской Америки, Таиланде, Испании или Португалии.

Для конкретной проверки этой гипотезы были определены различные показатели экономического развития (благосостояние, индустриализация, урбанизация и образование) и подсчитаны средние значения для стран англосаксонского мира, Европы и Латинской Америки, классифицированных как более или менее демократические.

В каждом случае средние благосостояние, степень индустриализации и урбанизации и уровень образования оказываются значительно выше в более демократических странах... <.. .>

За основные показатели благосостояния мы принимаем доход на душу населения, число человек на одно транспортное средство и на одного врача, а также число радиоприемников, телефонов и газет на 1000 человек. Различия поразительны во всех случаях. В более демократических европейских странах на одно транспортное средство приходится 17 человек, а в менее демократических странах - 143. В менее диктаторских латиноамериканских странах на одно моторное средство приходится 99 человек, тогда как в более диктаторских - 2741. Различия в доходах между группами тоже огромные:

1 Необходимо помнить, что эти цифры - средние, собранные из данных переписей для разных стран. Эти данные очень различаются по точности, и у нас нет способа измерить валидность сводных цифр, которые здесь представлены. Основным признаком валидности являются устойчивая направленность всех этих различий и их величина.

среднедушевые доходы падают с 695 долл. для более демократических стран Европы до 308 долл. для менее демократических стран; а соответствующие различия для Латинской Америки - от 171 до 119 долл. на душу населения. Тенденции тоже устойчивы: низший доход на душу населения в каждой группе приходится на «менее демократические» страны, высший - на «более демократические».

Индустриализация - а показатели благосостояния, разумеется, с ней связаны - измеряется процентной долей мужчин, занятых в сельском хозяйстве, и потребляемой в стране коммерчески производимой «энергией» в расчете на душу населения, т.е. в тоннах угля на человека в год. Оба этих показателя демонстрируют одинаково устойчивые результаты. Средняя доля работающих мужчин, занятых в сельском хозяйстве и смежных занятиях, составляла 21% в «более демократических» европейских странах, 41% - в «менее демократических», 52% - в «менее диктаторских» странах Латинской Америки и 67% - в «более диктаторских». Различия в потребляемой энергии на душу населения между странами столь же велики.

Степень урбанизации также связывается с наличием демократии1. Из данных, собранных в ходе Международного городского

1 Теоретики политики часто связывали урбанизацию с демократией. Гарольд Дж. Ласки утверждал, что «организованная демократия - продукт городской жизни» и что она, следовательно, естественным образом должна была «впервые эффективно проявить себя» в греческих городах-государствах, насколько бы ограниченным ни было их определение «гражданина». См. его статью «Демократия»: Lasky H.J. Democracy // Encyclopedia of the social sciences. - N.Y.: Macmillan, 1937. - Vol. 5. - P. 76-85. Макс Вебер утверждал, что город как определенный тип политического сообщества - специфически западный феномен, и проследил рождение понятия «гражданства» из социальных развитий, тесно связанных с урбанизацией. См. частичное изложение его точки зрения в главе, посвященной «городскому гражданству», в: Weber M. General economic history. - Glencoe (IL): Free press, 1950. - P. 315-338 (рус. пер.: Вебер М. История хозяйства // Вебер М. История хозяйства; Город. - М.: КАНОН-пресс-Ц: Кучково поле, 2001. - С. 286-305). Примечательно, что до 1933 г. наибольшей электоральной силой нацисты обладали в небольших сообществах и сельских районах. Берлин - единственный немецкий город с населением более 2 млн. человек - никогда не давал нацистам больше 25% голосов на свободных выборах. Типичный нацист, как и типичный французский пужадист или сегодняшний итальянский неофашист, был самозанятым жителем небольшого городка или сельского района. Хотя коммунисты как рабочая партия наиболее сильны в рабочих кварталах крупных городов в разных странах, они обладают высокой электоральной силой только в наименее урбанизированных европейских нациях, таких как Греция, Финляндия, Франция, Италия.

исследования (Беркли, шт. Калифорния), можно почерпнуть три показателя урбанизации: процент населения, проживающего в населенных пунктах с числом жителей 20 000 и более человек, процент населения, проживающего в сообществах численностью 100 000 и более человек, а также процентную долю населения, проживающего в стандартных метрополисных ареалах. В обоих исследуемых политико-культурных ареалах более демократические страны превосходят по всем трем показателям урбанизации страны менее демократические.

Многие предполагали, что чем население образованнее, тем больше шансов у демократии, и сравнительные данные, которыми мы располагаем, подтверждают это предположение. «Более демократические» страны Европы почти поголовно грамотны (наименьшая доля грамотных в них - 96%), тогда как в «менее демократических» нациях средняя доля грамотных в общей численности населении составляет 85%. В Латинской Америке эта разница следующая: средняя доля грамотных в «менее диктаторских» странах -74%, в «более диктаторских» - 46%1. Набор учащихся на 1000 человек на всех трех уровнях - начального, среднего и высшего образования - столь же устойчиво связан со степенью демократии. Чудовищное неравенство демонстрируется крайними случаями Гаити и Соединенных Штатов. В Гаити в начальной школе учится меньше детей (11 на 1000 человек), чем в США в колледжах (около 18 на 1000).

Связь между образованием и демократией заслуживает более широкого обсуждения, поскольку вся философия демократического правления видит в повышении уровня образованности распро-

1 Паттерн, находимый при сравнении средних для каждой группы стран, подтверждается амплитудами (верхними и нижними предельными значениями) по каждому показателю. Большинство амплитуд частично пересекаются, т.е. некоторые страны, относимые с точки зрения политики к низшей категории, располагаются по каким-то показателям выше, чем некоторые страны, находящиеся выше в шкале демократии. Обратим внимание на то, что как в Европе, так и в Латинской Америке нации, занимающие низшие позиции по любому из показателей, представленных в таблице [она здесь опущена. - Прим. пер.], оказываются также в категории «менее демократических». И наоборот, почти все страны, занимающие верхние позиции по любому из показателей, попадают в число «более демократических».

странение базовых требований демократии1. Как писал Брайс, обсуждая опыт Латинской Америки, «если образование и не делает людей хорошими гражданами, то, по крайней мере, с ним им легче стать таковыми»2. Образование предположительно расширяет кругозор людей, позволяет им понять необходимость норм толерантности, удерживает от приверженности экстремистским и монистическим доктринам и повышает их способность делать рациональные электоральные выборы.

В связи с индивидуальным поведением внутри стран данные, имеющие отношение к вкладу образования в демократию, выглядят даже нагляднее и убедительнее, чем в кросснациональных соотношениях. Данные, собранные службами исследований общественного мнения, которые опрашивали людей в разных странах на предмет их веры в демократические нормы терпимости к оппозиции, их отношения к этническим и расовым меньшинствам, а также об их вере в многопартийные системы, в противовес однопартийным, показали, что важнейшим фактором, отличающим тех, кто давал демократические ответы, от других людей, было образование. Чем выше у человека уровень образования, с тем большей вероятностью он верит в демократические ценности и поддерживает демократические практики3. Все соответствующие исследования показывают, что образование в этом плане гораздо важнее, чем доход или род занятий.

Это открытия должны побуждать нас ожидать гораздо более высокой корреляции между национальными уровнями образования и политической практикой, чем находимая фактически. Германия и Франция относились к числу наиболее образованных наций в Ев-

1 См.: Dewey J. Democracy and education. - N.Y.: Macmillan, 1916.

2

Цит. по: Whitaker A.P. The pathology of democracy in Latin America: A historian's point of view // American political science rev. - Cambridge, 1950. - Vol. 44, N 1. - P. 112. См. также: Mannheim K. Freedom, power and democratic planning. -N.Y.: Oxford univ. press, 1950.

См.: Smith C.H. Liberalism and level of information // J. of educational psychology. - Wash., 1948. - Vol. 39, N 2. - P. 65-82; Trow M.A. Right wing radicalism and political intolerance: PhD dissertation. - N.Y.: Columbia univ., 1957. - P. 17; Stouffer S. Communism, conformity and civil liberties. - N.Y.: Doubleday, 1955. -P. 138-139; Kido K., Suyi M. Report on social stratification and mobility in Tokyo // Japanese sociological rev. - Tokyo, 1954. - Vol. 4, N 1. - Pt. 3: The structure of social consciousness. - P. 74-100.

ропе, но сам по себе этот факт явно не стабилизировал их демократии. Может, однако, быть так, что образование помогало в сдерживании других антидемократических сил. Постнацистские данные из Германии ясно говорят о том, что более высокая образованность сопряжена с отказом от «сильной руки» и однопартийного прави-тельства1.

Хотя и нельзя сказать, что «высокий» уровень образования является для демократии достаточным условием, имеющиеся данные все же говорят о том, что он близок к тому, чтобы быть в современном мире необходимым ее условием. Так, если обратиться к Латинской Америке, где во многих странах до сих пор широко распространена неграмотность, то мы увидим, что из всех наций, в которых неграмотных больше половины населения, только одна -Бразилия - может быть включена в «более демократическую» группу.

У нас есть данные и из других экономически обездоленных культурных ареалов, свидетельствующие о том, что грамотность связана с демократией. Один из членов Лиги арабских государств, где после Второй мировой войны сохранялись демократические институты, а именно Ливан, является до сих пор самой образованной из арабских стран (грамотность там выше 80%). В остальной Азии к востоку от арабского мира всего лишь два государства -Филиппины и Япония - сохраняли после 1945 г. демократические режимы в отсутствие серьезных антидемократических партий. И эти две страны, хотя они и уступают по уровню дохода на душу населения любому европейскому государству, относятся к числу мировых лидеров в области образования. По доле людей, обучаю-

1 Дьюи предположил, что характер образовательной системы будет влиять на то, как она будет воздействовать на демократию, и это может частично пролить свет на источники нестабильности в Германии. Целью немецкого образования, согласно Дьюи (он писал в 1916 г.), была «скорее дисциплинарная выучка, чем... личностное развитие». Основной ее задачей было произвести «усвоение целей и смысла существующих институтов» и «глубокое подчинение» им. Это утверждение поднимает вопросы, которые мы не можем разобрать здесь подробно, но указывает на сложный характер связи между демократией и таким тесно связанным с ним фактором, как образование. (См.: Dewey J. Democracy and education. - N.Y.: Macmillan, 1916. - P. 108-110.) Оно предостерегает также от оптимистических выводов по поводу перспектив демократического развития в России, базирующихся на происходящей там сегодня колоссальной экспансии образования.

щихся в средней и высшей школе, Филиппины уступают фактически только Соединенным Штатам, а Япония превосходит по уровню образовательных достижений любое европейское государство1.

Хотя разные показатели были представлены раздельно, нам кажется очевидным, что факторы индустриализации, урбанизации, благосостояния и образования очень тесно взаимосвязаны и образуют единый общий фактор2. И эти факторы, подводимые под экономическое развитие, несут с собой в виде политического коррелята демократию3.

Прежде чем перейти к обсуждению внутренних связей между комплексом развития и демократией, можно упомянуть одно исследование Среднего Востока, которое своими существенными выводами подтверждает эти эмпирические связи применительно к другому культурному ареалу. Обследование шести стран Среднего Востока (Турции, Ливана, Египта, Сирии, Иордании и Ирана), проведенное Бюро прикладных социальных исследований Колумбийского университета в 1950-1951 гг., выявило значимые связи между урбанизацией, грамотностью, электоральной активностью, потреблением и производством медиа и образованием4. Были рас-

1 Цейлон, разделяющий с Филиппинами и Японией отличительную принадлежность к единственным демократическим странам Южной Азии и Дальнего Востока, в которых коммунисты лишены сколько-нибудь серьезного электорального веса, разделяет с ними и отличительную принадлежность к единственным странам этого ареала, где большинство населения является грамотным. Следует, однако, заметить, что на Цейлоне есть довольно большая троцкистская партия, являющаяся ныне официальной оппозицией; и хотя для Азии его образовательный уровень высок, он все же значительно ниже, чем у Японии или у Филиппин.

2 Это демонстрирует факторный анализ, проведенный на основе данных из 75 стран Лео Шнором (работа готовится к публикации).

3 Это утверждение «статистическое»; это с необходимостью означает, что из корреляций будет много исключений. Так, мы знаем, что в США и Англии менее состоятельные люди склонны голосовать, соответственно, за Демократическую или Лейбористскую партии. Тот факт, что значительное меньшинство низших слоев в этих странах голосует за более консервативную партию, не ставит под вопрос утверждение о том, что стратификационная позиция является основной детерминантой выбора партии в рамках многовариантного каузального процесса, заключенного в поведении людей или наций. Социальная наука, разумеется, никогда не сможет объяснить (предсказать) всего поведения.

4 См. об этом исследовании: Lerner D. The passing of traditional society. -Glencoe (IL): Free press, 1958. Корреляции получены из данных переписей; основ-

считаны простые и множественные корреляции между четырьмя базовыми переменными для всех стран, относительно которых имелась статистика ООН (в данном случае 54). Множественные корреляции при поочередном рассмотрении каждой из переменных как зависимой выглядят следующим образом1 (см. табл. 2):

Таблица 2

Зависимая переменная Коэффициент множественной корреляции

Урбанизация 0,61

Грамотность 0,91

Медиа-участие 0,84

Политическое участие 0,82

На Среднем Востоке Турция и Ливан превосходят по большинству этих показателей четыре другие проанализированные страны, и Лернер отмечает, что «великие послевоенные события в Египте, Сирии, Иордании и Иране были насильственной борьбой за контроль над властью - борьбой, которая примечательным образом отсутствует в Турции и Ливане, где вопросы контроля над властью решаются с помощью выборов»2.

Одним из научных вкладов Лернера является указание на то, какие последствия для целостной стабильности имеет диспропорциональное развитие в том или ином направлении, и на необходимость скоординированных изменений во всех этих переменных.

ные разделы обследования были посвящены реакциям на массмедиа и мнениям по поводу них, из чего делались выводы о типах личности, соответствующих современному и традиционному обществу.

1 Lerner D. The passing of traditional society... - P. 63. Показателем политического участия был процент проголосовавших на пяти последних выборах. Эти результаты нельзя рассматривать как независимую верификацию связей, представленных в этой статье, поскольку данные и переменные в основе своей те же самые (что и в работе: Shannon L. Is level of government related to capacity for self-government? // American j. of economics a. sociology. - N.Y., 1958. - Vol. 17, N 4. -P. 367-382), но тождественность результатов, полученных с помощью трех совершенно разных методов (фи-коэффициента, множественных корреляций и средних и амплитуд), определенно указывает на то, что эти связи нельзя приписать искусственным эффектам подсчетов. Следует также заметить, что эти три анализа были осуществлены так, что исследователи, их осуществлявшие, ничего о работе друг друга не знали.

Lerner D. The passing of traditional society. - Glencoe (IL): Free press, 1958. -

iНе можете найти то, что вам нужно? Попробуйте сервис подбора литературы.

P. 84-85.

Так, сравнивая урбанизацию и грамотность в Египте и Турции, он приходит к выводу о том, что хотя Египет гораздо более урбанизирован по сравнению с Турцией, он реально не «модернизирован» и даже не имеет адекватной основы для модернизации, поскольку отстает в грамотности. В Турции все показатели модернизации шли в ногу друг с другом: растущее участие в голосовании (36% в 1950 г.), растущая грамотность, урбанизация и т.д. По контрасту с этим города в Египте полны «бездомных неграмотных», которые становятся готовой аудиторией для политической мобилизации на поддержку экстремистских идеологий. Исходя из посылки функциональной взаимозависимости факторов «модернизации», Египет по шкале Лернера должен был бы быть вдвое грамотнее Турции, поскольку он вдвое больше урбанизирован. Тот факт, что он вдвое уступает ей в грамотности, объясняет, по мнению Лернера, те «дисбалансы», которые «имеют тенденцию входить в порочный круг и ускорять социальную дезорганизацию» - экономическую и политическую1.

Лернер вводит одно важное теоретическое добавление, предполагая, что эти ключевые переменные процесса модернизации можно рассматривать как исторические фазы и демократия является частью позднейшего развития, «завершающим институтом общества участия» (таков один из терминов, применяемых им для обозначения современного индустриального общества). Его точку

1 Lerner D. The passing of traditional society. - Glencoe (IL): Free press, 1958. -P. 87-89. Другие теории недоразвитых регионов тоже подчеркивали «круговой» характер сил, поддерживающих данный уровень экономического и социального развития; и в каком-то смысле эту мою статью можно рассматривать как попытку распространить анализ комплекса институтов, конституирующих «модернизированное общество», на политическую сферу. В неопубликованной монографии Лео Шнора «Экономическое развитие и урбанизация: Экологический подход» технологические, демографические и организационные (включая грамотность и доход на душу населения) переменные связываются во взаимозависимый комплекс. В недавно опубликованной книге Харви Либенстайна «Экономическая отсталость и экономический рост» «недоразвитость» рассматривается в терминах экономической теории «квазиравновесия» как комплекс связанных и взаимно поддерживающих друг друга аспектов общества, и в этот комплекс включаются культурные и политические характеристики - неграмотность, отсутствие среднего класса, примитивные системы коммуникаций (Liebenstein H. Economic backwardness and economic growth. - N.Y.: Wiley, 1957. - P. 39-41).

зрения на связи между этими переменными, рассматриваемыми как стадии, стоит процитировать подробнее:

«Длительная эволюция общества участия, судя по всему, заключает в себе регулярную последовательность из трех фаз. Сначала приходит урбанизация, ибо только в городах развивался комплекс умений и ресурсов, характеризующий современную индустриальную экономику. Внутри этой городской матрицы развиваются оба атрибута, определяющих своеобразие следующих двух фаз: грамотность и развитие медиа. Между ними существует тесная взаимная связь, ибо грамотные люди развивают медиа, а те в свою очередь ведут к распространению грамотности. При этом ключевую функцию во второй фазе выполняет все-таки грамотность. Способность читать, приобретаемая поначалу лишь относительно немногими, оснащает их для выполнения разных задач, требуемых модернизирующимся обществом. И только в третьей фазе, когда совершает рывок вперед сложная технология промышленного развития, общество начинает в массовом масштабе производить газеты, сети радиовещания и кино. Это в свою очередь ускоряет распространение грамотности. Из этого взаимодействия и развиваются институты участия (например, голосования), которые мы находим во всех развитых современных обществах»1.

1 Lerner D. The passing of traditional society. - Glencoe (IL): Free press, 1958. -P. 60. Лернер сосредоточивается также на некоторых личностных требованиях «современного» общества, которые также можно связать с личностными требованиями демократии. Согласно ему, физическая и социальная мобильность современного общества требует мобильной личности, способной адаптироваться к быстрому изменению. Развитие «мобильной восприимчивости, столь приспособленной к изменению, что отличительной ее особенностью становится переупорядочение системы Я», было делом ХХ в. Главная особенность такой личности - это эмпа-тия, под которой понимается «общая способность видеть себя в ситуации другого, неважно в лучшем или худшем свете» (Lerner D. The passing of traditional society... - P. 49 ff). Приносит ли эта психологическая характеристика в итоге предрасположенность к демократии (подразумевающую готовность к принятию точки зрения другого), или она связывается с антидемократическими склонностями личностного типа, развивающегося в «массовом обществе» (предполагающего отсутствие сколько-нибудь прочных личностных ценностей, укорененных в вознаграждающем участии) - вопрос открытый. Возможно, эмпатия, или более или менее «космополитический» взгляд на вещи, является общей личностной характеристикой современных обществ, а другие особые условия определяют, будет ли она иметь в виде социальных последствий толерантность и демократические установки или отсутствие укорененности и аномию.

Тезис Лернера о функциональной взаимозависимости этих элементов модернизации никоим образом не устанавливается его данными, но материал данной статьи дает возможность провести изыскания в этом направлении. Отклоняющиеся случаи вроде Египта, где «отстающая» грамотность связана с серьезными напряжениями и потенциальными беспорядками, можно найти также в Европе и в Латинской Америке, и их анализ - не входящий здесь в наши задачи - лучше прояснит базовую динамику модернизации и проблему социальной стабильности в потоке институционального изменения.

Несколько процессов подчеркивают эти корреляции, наблюдаемые во многих ареалах мира, впридачу к уже обсуждавшемуся ранее воздействию высокого уровня образования и грамотности на создание и поддержание веры в демократические нормы. Наиболее важна здесь, пожалуй, связь между модернизацией и формой «классовой борьбы». Если взять низшие слои, то экономическое развитие, означающее рост дохода, растущую экономическую обеспеченность и более высокий уровень образования, позволяет людям в этом статусе развивать дальновидность и более сложные и умеренные взгляды на политику. Вера в длительные постепенные реформы может быть идеологией только относительно обеспеченного низшего класса1. Растущие благосостояние и образование способствуют демократии еще и тем, что повышают степень, в которой низшие слои сталкиваются с перекрестными давлениями, снижающими силу их приверженности конкретным идеологиям и уменьшающими их восприимчивость к экстремистским идеологиям. Как протекает этот процесс, мы рассмотрим подробнее во второй части статьи, но если сказать самое главное, то функционирует он посредством расширения их вовлеченности в интегрированную национальную культуру (отличную от обособленной культуры низшего класса) и, следовательно, увеличения их контакта с ценностями среднего класса. Маркс утверждал, что пролетариат есть революционная сила, поскольку ему нечего терять, кроме своих цепей, приобрести же он может весь мир. Токвиль, анализируя, почему низшие классы в Америке поддерживают систему, пере-

1 Cm. : Lipset S.M. Socialism-East and West-Left and Right // Confluence. -South Bend (IN), 1958. - Vol. 7, N 2. - P. 173-192.

фразировал Маркса еще до того, как тот проделал свой анализ; он заметил, что «бунтуют лишь те, кому нечего терять»1.

Рост благосостояния не только каузально связан с развитием демократии через изменение социальных условий рабочих, но и воздействует на политическую роль среднего класса, изменяя форму стратификационной структуры таким образом, что она превращается из вытянутой пирамиды с огромным низшим классом, находящимся в ее основании, в ромб с растущим средним классом. Обширный средний класс играет смягчающую роль в модерирова-нии конфликтов, поскольку способен вознаграждать умеренные и демократические партии и наказывать экстремистские группы.

Национальный доход связан также с политическими ценностями и стилем высшего класса. Чем беднее страна и чем ниже абсолютный стандарт жизни низших классов, тем большее давление испытывают высшие слои к тому, чтобы считать низшие классы находящимися вне рамок человеческого общества, вульгарными, от природы низшими, низшей кастой. Разительное различие между верхушкой и социальным дном в стиле жизни делает это психологически неизбежным. Следовательно, высшие слои также склонны считать предоставление низшим слоям политических прав, особенно права причастности к власти, по существу абсурдным и аморальным. Верхние слои не только сами сопротивляются демократии, но и зачастую их надменное политическое поведение способствует усилению экстремистских реакций со стороны низших классов.

Общий уровень дохода нации тоже будет оказывать воздействие на ее восприимчивость к демократическим нормам политической толерантности. Ценности, предполагающие, что не имеет особого значения, какая из сторон правит, и что ошибку можно стерпеть даже от правящей партии, лучше всего развиваются там, где: а) у правительства не так уж много полномочий, чтобы влиять на ключевые жизненные шансы большинства властных групп, или б) в стране достаточно богатств, чтобы действительно не имело большого значения некоторое их перераспределение. Если утрата постов означает для основных властных групп серьезную потерю,

1 Tocqueville A. de. Democracy in America. - N.Y.: Knopf Vintage, 1945. -Vol. 1. - P. 258.

то они будут с большей готовностью прибегать к радикальным мерам в попытке сохранить или заполучить эти посты. Уровень благосостояния будет также влиять на то, в какой степени те или иные страны смогут развить среди своих гражданских служащих и политиков «универсалистские» нормы (отбор на основе компетентности, исполнение обязанностей без фаворитизма). Чем беднее страна, тем сильнее выражен непотизм, т.е. поддержка родственников и друзей. Слабость универсалистских норм уменьшает возможность развития эффективной бюрократии, необходимой для современного демократического государства1.

С ростом благосостояния не столь непосредственно, но все же, видимо, связано наличие посреднических организаций и институтов, которые могут выступать источниками уравновешивающей силы и рекрутерами участников в политический процесс таким образом, как об этом говорится у Токвиля и у других приверженцев того, что получило известность как теория «массового общества»2. Они утверждали, что общество, в котором нет многочисленных организаций, относительно независимых от центральной государственной власти, имеет высокий диктаторский и революционный потенциал. Такие организации выполняют ряд функций, необходимых для демократии: они являются источником уравновешивающей силы, не дающей государству или любому единичному источнику частной власти установить господство над всеми политическими ресурсами; они служат источником новых мнений; они могут быть средством передачи идей, особенно оппозиционных, для большой части граждан; они помогают прививать людям навыки политики; и они способствуют повышению уровня интереса к политике и участия в ней. Хотя надежных данных, касающихся связи между

1 Обсуждение проявлений этой проблемы в новом государстве см.: Apter D. The Gold Coast in transition. - Princeton (NJ): Princeton univ. press, 1955 (особенно гл. 9, 13). Аптер показывает важность эффективной бюрократии и принятия бюрократических ценностей и поведенческих паттернов для существования демократического политического порядка.

См.: Lederer E. The state of the masses. - N.Y.: Norton, 1940; Arendt H. Origins of totalitarianism. - N.Y.: Harcourt Brace, 1950; Horkheimer M. Eclipse of reason. -N.Y.: Oxford univ. press, 1947; Mannheim K. Man and society in an age of reconstruction. - N.Y.: Harcourt Brace, 1940; Selznick P. The organizational weapon. - N.Y.: McGraw-Hill, 1952; Ortega y Gasset J. The revolt of the masses. - N.Y.: Norton, 1932.

национальными формами добровольных организаций и национальными политическими системами, у нас нет, сведения, полученные из исследований индивидуального поведения в нескольких разных странах, говорят о том, что, вне зависимости от других факторов, люди, принадлежащие к ассоциациям, чаще придерживаются демократических мнений по вопросам, относящимся к толерантности и партийным системам, и с большей вероятностью участвуют в политическом процессе как активисты или как избиратели. Поскольку мы знаем также, что чем состоятельнее человек и чем лучше он образован, тем выше вероятность того, что он будет принадлежать к добровольным организациям, то представляется вероятным, что склонность к образованию таких групп есть функция уровня дохода и возможностей досуга внутри данной нации1.

1 См.: Banfield E. The moral basis of a backward society. - Glencoe (IL): Free press, 1958. В этой книге содержится великолепное описание того, как глубокая нищета препятствует развитию коммунистических организаций в южной Италии. Данные, полученные в ходе опросов общественного мнения в США, Германии, Франции, Великобритании и Швеции, показывают, что где-то от 40 до 50% взрослых в этих странах участвуют в добровольных ассоциациях, и в менее стабильных демократиях, Франции и Германии, уровни членства в них не ниже, чем в более стабильных - США, Великобритании и Швеции. Похоже, эти результаты ставят под вопрос наше общее положение, но никакого определенного вывода сделать нельзя, поскольку в большинстве исследований использовались несопоставимые категории. Это требует проведения дальнейших исследований во многих странах. Данные об этих странах см. в следующих исследованиях. О Франции: Rose A. Theory and method in the social sciences. - Minneapolis: Univ. of Minnesota press, 1954. - P. 74; Gallagher O.R. Voluntary associations in France // Social forces. -Chapel Hill (NC), 1957. - Vol. 36, N 2. - P. 154-156. О Германии: Reigrotski E. Soziale Verflechtungen in der Bundesrepublik. - Tübingen: Mohr, 1956. - S. 164. О США: Wright C.R., Hyman H.H. Voluntary association memberships of American adults: Evidence from national sample surveys // American sociological rev. - Wash., 1958. -Vol. 23, N 3. - P. 287; Scott J.C., jr. Membership and participation in voluntary associations // American sociological rev. - Cambridge, 1957. - Vol. 22, N 3. - P. 315-326; Maccoby H. The differential political activity of participants in a voluntary association // American sociological rev. - Cambridge, 1958. - Vol. 23, N 5. - P. 524-533. О Великобритании см.: Puzzled people: A study in popular attitudes to religion, ethics, progress and politics in a London borough: Mass observation. - L.: Gollanz, 1947. -P. 119; Bottomore T. Social stratification in voluntary organizations // Social mobility in Britain / Ed. by D. Glass. - Glencoe (IL): Free press, 1954. - P. 354. О Швеции см.: Heckscher G. Pluralist democracy: The Swedish experience // Social research. - N.Y., 1948. - Vol. 15, N 4. - P. 417-461.

Очевидно, что демократия и обсуждаемые здесь условия, связанные со стабильной демократией, локализованы, по сути, в странах северо-западной Европы и в их англоязычном потомстве в Америке и Австралии. Макс Вебер, среди прочих, утверждал, что факторы, способствующие демократии в этом ареале, представляют собой исторически уникальное сцепление элементов и часть комплекса, произведшего в этом же ареале капитализм. Основной его довод состоял в том, что капиталистическое хозяйственное развитие (которое находит поддержку и лучше всего продвигается в протестантских странах) создало класс бюргеров, существование которого было как катализатором, так и необходимым условием демократии. Особый акцент, придаваемый в протестантизме индивидуальной ответственности, способствовал появлению демократических ценностей. Большая исходная сила средних классов в этих странах имела следствием альянс между бюргерами и троном, и этот альянс сохранял монархию, способствуя тем самым легитимации демократии среди консервативных слоев. Таким образом, мы находим здесь взаимосвязанный кластер экономического развития, протестантизма, монархии, постепенного политического изменения, легитимности и демократии1. Можно спорить о том, является ли какой-то из аспектов в этом кластере первичным, но мы имеем здесь именно кластер связанных друг с другом факторов и сил.

1 Вводя исторические события как часть анализа факторов, внешних для политической системы и составляющих часть каузального нексуса, в который вплетена демократия, я действую в русле хорошей социологической и даже функ-ционалистской традиции. Как хорошо сказал Рэдклифф-Браун, «одним из "объяснений" социальной системы будет ее история, если она нам известна, т.е. подробное описание того, как она сложилась, что она собой представляет и где она находится. Другое "объяснение" той же системы достигается путем демонстрации того. что она представляет собой особое проявление законов социальной психологии или социального функционирования. Эти два вида объяснения не противоречат друг другу, они друг друга дополняют» (Radcliffe-Brown A.R. On the concept of function in social science // American anthropologist. - Arlington (VA), 1935. -Vol. 37, N 3. - P. 401). См. также: Weber M. The methodology of the social sciences. -Glencoe (IL): Free press, 1949. - P. 164-188, где подробно обсуждается роль исторического анализа в социологическом исследовании.

III. Легитимность и демократия

В этой части я обращусь к рассмотрению некоторых предпосылок демократии, вытекающих из специфически исторических элементов этого комплекса, особенно тех, которые связаны с потребностью демократической политической системы в легитимности и в механизмах, снижающих остроту политического раскола. Эти предпосылки соотносятся с экономическим развитием, но в то же время отличны от него, так как являются элементами самой политической системы.

Легитимность и эффективность. В современном мире, как мы попытались показать в предыдущем разделе статьи, экономическое развитие, предполагающее индустриализацию, урбанизацию, высокие образовательные стандарты и устойчивый рост совокупного богатства общества, есть базовое условие поддержания демократии; это признак эффективности всей системы.

Между тем стабильность данной демократической системы зависит не только от эффективности системы в модернизации, но и от эффективности и легитимности политической системы. Под эффективностью понимается действительная работа политической системы, т. е. степень удовлетворения ею основных управленческих функций, как они определяются ожиданиями большинства членов общества и ожиданиями могущественных групп внутри него, которые могли бы угрожать системе, таких как вооруженные силы. Эффективность демократической политической системы, характеризуемую эффективной бюрократией и системой принятия решений, способной решать политические проблемы, можно отличать от эффективности всей системы, хотя сбой в функционировании общества в целом будет, конечно, оказывать воздействие на политическую подсистему. Легитимность подразумевает способность политической системы порождать и поддерживать веру в то, что существующие политические институты являются наиболее подходящими и приемлемыми для общества. Степень легитимности существующих сегодня демократических политических систем во многом зависит от того, как ими решены ключевые проблемы, исторически разделявшие общество. Задача этого раздела статьи состоит в том, чтобы, во-первых, показать, как степень легитимности демократической системы может влиять на ее способность пережи-

вать кризисы эффективности, такие как депрессии или поражения в войнах, и, во-вторых, указать на способы, которыми разные урегулирования базовых исторических расколов - а ими определяется легитимность разных систем - укрепляют или ослабляют демократию, воздействуя на нынешнюю партийную борьбу.

Если эффективность - прежде всего инструментальный параметр, то легитимность - в большей степени аффективный и оценочный. Группы будут считать политическую систему легитимной или нелегитимной в зависимости от того, как будут сочетаться ее ценности с их первичными ценностями. Важные сегменты армии, гражданской службы и аристократических классов в Германии отвергли Веймарскую республику не из-за ее неэффективности, а потому что ее символы и базовые ценности отрицали их символы и ценности. Легитимность сама по себе соединима со многими формами политической организации, в том числе репрессивными. До индустриализации феодальные общества, несомненно, пользовались базовой лояльностью большинства своих членов. Кризисы легитимности появились как исторический феномен недавно, вслед за развитием острых расколов между группами, сумевшими благодаря ресурсам массовой коммуникации организоваться вокруг иных ценностей, нежели те, которые считались прежде единственно легитимными для всего общества.

Кризис легитимности - это кризис изменения, и, следовательно, корни его как фактора, влияющего на стабильность демократических систем, нужно искать в характере изменений в современном обществе. Можно выдвинуть гипотезу о том, что кризисы легитимности происходят во время перехода к новой социальной структуре, а) если на раннем этапе переходного периода или по крайней мере тогда, когда все основные группы развивают политические требования, этим группам не обеспечен доступ к политической системе; или б) если в период структурного изменения ставится под угрозу статус основных консервативных институтов. Когда установилась новая социальная структура, новый кризис может развиться в случае, если новая система не сможет отвечать ожиданиям основных групп (на основе «эффективности») достаточно долго, чтобы сложилась легитимность на этой новой основе.

Токвиль образно описал первый общий тип потери легитимности, имея в виду прежде всего страны, перешедшие от аристокра-

тических монархий к демократическим республикам: «... бывают в жизни нации эпохи, когда старые обычаи народа меняются, общественная нравственность разрушается, религиозная вера оказывается поколебленной и чары традиции рушатся.». В эти эпохи граждане не имеют «ни инстинктивного патриотизма монархии, ни рассудочного патриотизма республики. они останавливаются посередине между тем и другим, погружаясь в замешательство и бедствие»1.

Но если статусу основных консервативных групп и символов в этот переходный период ничто не угрожает, то, даже если они и теряют при этом большую часть своей власти, демократия, видимо, гораздо более защищена. Красноречивым свидетельством связи между сохраняющейся легитимностью консервативных институтов и демократией является связь между монархией и демократией. В свете того, какую роль сыграли в порождении современных демократических политических движений американская и французская республиканские революции, тот факт, что 10 из 12 стабильных европейских и англоязычных демократий являются монархиями, представляется весьма любопытной корреляцией. Великобритания, Швеция, Норвегия, Дания, Нидерланды, Бельгия, Люксембург, Австралия, Канада и Новая Зеландия - королевства; а единственными республиками, отвечающими двум условиям - а именно стабильности демократических процедур со времени утверждения демократии и отсутствию крупных тоталитарных движений за последние 25 лет, - являются США, Швейцария и Уругвай. Нации, которые переходили от абсолютизма и олигархии (связанных с государственной церковью) к демократическому государству благосостояния, сохраняя в той или иной форме монархию, видимо, чаще способны проводить изменения, удерживая непрерывную нить

2

легитимности в своих политических институтах .

1 Tocqueville A. de. Democracy in America. - N.Y.: Knopf Vintage, 1945. -Vol. 1. - P. 251-252.

Уолтер Липпман, говоря о, видимо, большей способности конституционных монархий, по сравнению с европейскими республиками, «сохранять порядок вместе со свободой», предполагает, что это может объясняться тем, что «в республике верховная власть, будучи полностью секуляризованной, теряет значительную часть своего престижа; если угодно, она очищается от всякой видимости внутреннего величия». См.: Lippman W. The public philosophy. - N.Y.: Mentor books, 1956. -P. 50.

Сохранение монархии, несомненно, сберегало для системы лояльность аристократических, традиционалистских и клерикальных частей населения, возмущенных растущей демократизацией и эгалитаризмом. И одновременно, благодаря более великодушному принятию низших слоев и воздержанию от такого сопротивления, которое могло бы вызвать необходимость революции, консервативные порядки завоевывали или удерживали лояльность новых «граждан». Там, где монархия опрокидывалась революцией и упорядоченный ход событий прерывался, силы, связанные с монархией, иногда продолжали отказывать преемникам-республиканцам в легитимности на протяжении пяти поколений и больше.

Единственная конституционная монархия, которая стала фашистской диктатурой, - Италия - была, как и Французская республика, относительно новой и еще нелегитимной для большинства групп в обществе. Савойский дом оттолкнул от себя католиков, разрушив светскую власть пап, и не был легитимным преемником в старом Королевстве Обеих Сицилий. Вплоть до Первой мировой войны церковь по сути запрещала католикам участвовать в итальянской политике, и запрет этот был отменен ею только из страха перед социалистами. Схожая установка была принята в то же время французскими католиками в отношении Третьей республики. И итальянской, и французской демократии приходилось на протяжении большей части истории обходиться без лояльной поддержки со стороны важных групп в их обществе - как слева, так и справа. Таким образом, один из основных источников легитимности заключен в преемственности первичных консервативных и интегра-тивных институтов в течение переходного периода, в котором рождаются новые социальные институты.

Второй общий тип потери легитимности, как уже отмечалось, связан с тем, как общества справляются с проблемой «вхождения в политику». Определение того, когда новые социальные группы получат доступ к политическому процессу, влияет на легитимность политической системы для консервативных или рождающихся новых групп. В XIX в. этими новыми группами были прежде всего промышленные рабочие. В кризис «вхождения в политику» ХХ в., как правило, оказываются вовлечены колониальные элиты и крестьянские народы. Когда бы новые группы ни становились политически активными (например, когда рабочие впервые

ищут доступ к экономической и политической власти через экономическую организацию и избирательное право, когда буржуазия требует доступа к государственному управлению и участию в нем или когда колониальные элиты требуют контроля над своей системой), сравнительно легкий доступ к легитимным политическим институтам обычно приносит системе лояльность новых групп, и они, в свою очередь, могут просачиваться в старые господствующие слои, сохраняя тем самым их статусную цельность. В таких нациях, как Германия, где в доступе долгое время отказывалось сначала буржуазии, а позднее рабочим, и где для ограничения доступа использовалась сила, низшие слои отчуждались от системы и подталкивались к принятию экстремистских идеологий, которые в свою очередь отчуждали более утвердившиеся группы от принятия рабочего политического движения как легитимной альтернативы.

Политические системы, не позволявшие новым слоям получить доступ к власти иначе, нежели революционными средствами, также тормозили рост легитимности через выведение на политическую арену вековых чаяний. Группы, чувствующие себя обязанными пробиться в политический аппарат силовыми средствами, обычно преувеличивают возможности, открываемые политическим участием. Они надеются на гораздо большее, чем допускается внутренними ограничениями политической стабильности. Следовательно, демократические режимы, родившиеся под таким давлением, будут не только сталкиваться с той проблемой, что их будут считать нелегитимными группы, лояльные ancient regime, но и могут быть отвергнуты теми, чьи вековые чаяния не будут удовлетворены свершившимся изменением. Пример такого феномена дает, видимо, Франция. Правые клерикалы считали Республику нелегитимной, а части низшего класса нетерпеливо ждали исполнения своих вековых надежд. Многие из обретших независимость наций Азии и Африки столкнулись с проблемой обеспечения лояльности масс демократическим государствам, мало что способным сделать для достижения тех утопических целей, которые были поставлены националистическими движениями в эпоху колониализма и в переходный период борьбы за независимость.

Итак, мы обсудили несколько условий, влияющих на сохранение или исходное обеспечение легитимности политической системы. При наличии приемлемой эффективности в случае, если что-то уг-

рожает статусу основных консервативных групп или если в решающие периоды доступ к политической системе окажется перекрытым, легитимность системы будет оставаться под вопросом. Даже в легитимных системах падение эффективности, регулярно повторяющееся или продолжительное, будет угрожать их стабильности.

Важным показателем легитимности является степень, в которой данная нация развила общую «секулярную политическую культуру», национальные ритуалы и праздники, служащие поддержанию легитимности различных демократических практик1. США развили общую гомогенную секулярную политическую культуру, находящую выражение в поклонении и консенсусе, окружающих фигуры Отцов-основателей, Джефферсона, Линкольна, Теодора Рузвельта и их принципы. Эти общие элементы, к которым апеллируют все американские политики, имеются не во всех демократических обществах. В некоторых европейских странах левые и правые обладают разными наборами символов и историческими политическими героями. Франция представляет ярчайший пример нации, не развившей такого общего наследия. Так, многие битвы с использованием разных символов, происходившие между левыми и правыми с 1789 г. и в течение почти всего XIX в., «все еще продолжаются, и проблемы остаются открытыми; каждая из этих дат [крупных политических споров] до сих пор разделяет левых и правых, клерикалов и антиклерикалов, прогрессистов и реакционеров во всех их исторически детерминированных констелляциях»2.

Как мы увидели, нации могут очень отличаться друг от друга в том, в какой степени их политические институты рассматриваются разными слоями как легитимные. Знание относительной степени легитимности политических институтов конкретной нации имеет ключевое значение для любой попытки проанализировать стабильность этих институтов в случае их столкновения с кризисом эффективности. Связь между разными степенями легитимности и эффективности в конкретных политических системах может быть нагляднее представлена в виде табл. 3, с примерами стран, характеризуемых четырьмя отраженными в ней возможными комбинациями.

1 Cm.: Almond G. Comparative political systems // J. of politics. - Cambridge, 1956. - Vol. 18, N 3. - P. 391-409.

2

Luethy H. The state of France. - L.: Secker & Warburg, 1955. - P. 29.

Таблица 3

ЭФФЕКТИВНОСТЬ

+ -

ЛЕГИТИМНОСТЬ + A B

- C D

Общества, попадающие в ячейку А, т.е. те, которые обладают высокой легитимностью и эффективностью, будут, несомненно, иметь стабильные политические системы. Такие государства, как США, Швеция и Британия, соответствуют базовым политическим потребностям своих граждан, имеют эффективные бюрократии и системы принятия политических решений, обладают традиционной легитимностью благодаря долгой преемственности ключевых символов суверенитета, монархии или конституции и не содержат сколько-нибудь важных меньшинств, базовые ценности которых расходятся с базовыми ценностями системы1. Неэффективные и нелегитимные режимы, находимые в ячейке Э, должны быть, по определению, нестабильными и терпеть крах, если только это не диктатуры, поддерживающие себя силой, как, например, сегодняшние правительства Венгрии и Восточной Германии. Политический опыт разных стран начала 1930-х годов иллюстрирует следствия разных комбинаций легитимности и эффективности. В конце 1920-х годов ни в Германии, ни в Австрии республики не рассматривались значительными и могущественными сегментами населения как легитимные, однако оставались приемлемо эффективны-ми2. В таблице они попадают в ячейку С.

1 Расовая проблема на американском Юге создает одну из основных угроз легитимности системы и одно время была причиной крушения национального порядка. Этот конфликт до сих пор ослабляет приверженность многих белых южан демократическим правилам. В Великобритании схожая проблема существовала до тех пор, пока частью Соединенного Королевства оставалась католическая Ирландия. Эффективность правительства не могла удовлетворить Ирландию. Политические практики обеих сторон конфликта в Северной Ирландии (Ольстере) также иллюстрируют проблему режима, который для значительного сегмента населения остается нелегитимным.

Великолепный анализ перманентного кризиса Австрийской республики, вытекавшего из того, что ее признавали нелегитимным режимом католики и консерваторы, см. в книге: Gulick C. Austria from Hapsburg to Hitler. - Berkeley: Univ. of California press, 1948.

Когда эффективность правительств разных стран в 1930-е годы упала, те общества, которые обладали высоким уровнем легитимности, оставались демократическими, а страны с низкой легитимностью, такие как Германия, Австрия и Испания, потеряли свою свободу, и эта же участь чуть не постигла Францию. Если представить суть этих перемен в терминах местонахождений в нашей таблице, то страны, переходившие из категории A в категорию B, оставались демократическими, а политические системы стран, переходивших из категории C в категорию D, терпели крах. И оставалось только случиться военному поражению 1940 г., чтобы убедительно доказать низкую позицию французской демократии по шкале легитимности. Это была единственная низвергнутая демократия, которая оказала масштабную поддержку режиму Квислинга1.

Ситуации, подобные только что рассмотренным, в которых какой-то из двух параметров - легитимность или эффективность -имеет высокий уровень, а другой - низкий, демонстрируют полезность этого типа анализа. В краткосрочной перспективе высокоэффективная, но нелегитимная система (как, например, хорошо управляемая колония) менее стабильна, чем режимы с относительно низкой эффективностью, но высоким уровнем легитимности. Социальная стабильность такой нации, как Таиланд, - несмотря на все coups d'états, которые там время от времени случаются, - находится в разительном контрасте с ситуацией в соседних нациях Юго-Восточной Азии, бывших в прошлом колониями. Связь между анализом легитимности и обсуждением вклада экономического развития в демократию, предпринятым ранее, очень хорошо видна в процессах, посредством которых режимы с низким уровнем легитимности обретают эту легитимность, и, наоборот, в процессах,

1 Проблему легитимности во Франции хорошо описала Катрин Манро: «Правые партии никогда не забывали до конца о возможности контрреволюции, а левые возрождали Революцию, воинствуя в своем марксизме или коммунизме; каждая сторона подозревала другую в использовании Республики для достижения собственных целей и в том, что та лояльна Республике лишь постольку, поскольку ей это выгодно. Время от времени эта подозрительность грозила тем, что Республика станет неработоспособной, поскольку вела к обструкции в политической и экономической сфере, а трудности в управлении, в свою очередь, подрывали уверенность в режиме и его правителях». Цит. по: Micaud C.A. French political parties: Ideological myths and social realities // Modern political parties / Ed. by S. Neumann. -Chicago (IL): Univ. of Chicago press, 1956. - P. 108.

которые связаны с коллапсом легитимной системы. Продолжительная эффективность, сохраняющаяся в течение нескольких поколений, может давать политической системе легитимность; в современном мире такая эффективность в основном означает постоянное экономическое развитие. Так, нации, наиболее успешно адаптировавшиеся к требованиям промышленной системы, были наименее подвержены внутренним политическим напряжениям и либо сохраняли свою традиционную легитимность, монархию, либо развивали новые сильные символы легитимности.

Социальная и экономическая структура, которую Латинская Америка унаследовала с Иберийского полуострова, помешала ей последовать по пути бывших английских колоний, и ее республики так никогда и не развили символов и ауры легитимности. Выживание новых политических демократий Азии и Африки в значительной мере связано с их способностью удержать продолжительный период эффективности, т.е. способностью удовлетворять определенные инструментальные потребности их населений.

Легитимность и раскол. Продолжительная эффективность системы в целом может, как в случае Соединенных Штатов и Швейцарии, со временем легитимировать демократическую политическую систему. Вместе с тем всем демократическим системам присуща постоянная угроза того, что конфликты между разными группами, составляющие источник жизненной силы систем, могут кристаллизоваться настолько, что возникнет опасность дезинтеграции общества. Поэтому к ключевым реквизитам демократической политической системы относятся, наряду с эффективностью, условия, служащие смягчению интенсивности партийной борьбы.

Поскольку наличие умеренного состояния конфликта является неотъемлемым аспектом легитимной демократической системы и фактически еще одним способом ее определения, то нас не должно удивлять, что главные факторы, определяющие такое оптимальное состояние, тесно связаны с теми, которые производят легитимность, рассматриваемую через преемственность символов и статуса. В сущности, характер и содержание основных расколов, воздействующих на политическую стабильность общества, в значительной степени определяются историческими факторами, повлиявшими на то, как были решены или надолго оставлены нерешенными основные проблемы, разделявшие общество.

В современную эпоху в западных государствах возникли три основные проблемы. Первой была проблема религиозная: место церкви и / или разных религий в пределах нации. Второй была проблема допуска низших слоев, особенно рабочих, к «гражданству», установления доступа к власти через всеобщее избирательное право и легитимного права коллективного торга в экономической сфере. Третьей проблемой была непрерывная борьба за распределение национального дохода.

Здесь важен следующий общий вопрос: подходили ли к этим главным проблемам по очереди, так что каждая оказывалась более или менее решена к тому моменту, как возникала следующая, или эти проблемы накапливались и исторически уже сложившиеся проблемы и источники раскола смешивались с новыми? Поочередное разрешение напряжений вносит вклад в стабильную политическую систему; перенос проблем из одного исторического периода в другой обусловливает политическую атмосферу, характеризующуюся скорее горечью и фрустрацией, чем терпимостью и компромиссом. Люди и партии начинают расходиться друг с другом не просто в способах решения текущих проблем, но и в фундаментальных и противостоящих weltanschauungen. Они начинают видеть в политической победе своих оппонентов значительную моральную угрозу; в итоге всей системе недостает эффективной ценностной интеграции.

Религиозная проблема - проблема места церкви в обществе -решалась и была решена в большинстве протестантских наций в XVIII-XIX вв. и с тех пор перестала быть предметом серьезных политических споров. В некоторых государствах, таких как США, церковь была отделена от государства и приняла этот результат. В других, таких как Британия, страны Скандинавии и Швейцария, религия сохраняет поддержку со стороны государства, но государственные церкви, как и конституционные монархи, обладают лишь номинальной властью и перестали быть важными источниками разногласий. Осталось высказаться о католических странах Европы, которые дают нам примеры ситуаций, в которых исторические споры между клерикальными и антиклерикальными силами, воспламененные Французской революцией, по сей день продолжают политически разделять людей. В таких странах, как Франция, Италия, Испания и Австрия, быть католиком означает быть в союзе с

правыми и консервативными группами в политике, а быть антиклерикалом (или членом религиозного меньшинства) означает чаще всего союз с левыми. В некоторых из этих стран новые проблемы, возникая, накладывались на религиозный вопрос; для консервативных католиков борьба с социалистами была не просто экономической борьбой или дебатами вокруг социальных институтов, а глубоко укорененным конфликтом между Богом и Сатаной, между добром и злом1. Для многих мирских интеллектуалов в сегодняшней Италии оппозиция церкви легитимирует альянс с коммунистами. До тех пор пока религиозные узы укрепляют мирские политические группирования, шансы на закрепление демократических взаимных уступок и компромиссов остаются слабыми.

Проблема «гражданства», или «политического равенства», тоже решалась по-разному. Так, США и Британия в начале и середине XIX в. дали гражданство рабочим. Швеция и ряд европейских наций противились этому вплоть до начала ХХ в., и борьба за гражданство оказалась соединена в этих странах с социализмом как политическим движением, порождая тем самым революционный

1 Связку между нестабильностью демократии и католицизмом можно объяснить также элементами, свойственными католицизму как религиозной системе. Демократия требует универсалистской системы политических верований, поскольку она легитимирует разные идеологии. И можно предположить, что религиозные ценностные системы, являющиеся более универсалистскими в смысле меньшего акцентирования претензий на статус единственной истинной церкви, будут более совместимы с демократией, чем те, которые полагают себя единственными носителями истины. Последнее представление, которого католики придерживаются гораздо сильнее, нежели большинство других христианских церквей, не позволяет религиозной ценностной системе содействовать легитимации политической системы, требующей - как части своей базовой системы ценностей -веры в то, что «благу» больше всего содействует конфликт между противостоящими мнениями.

Кингсли Дэвис утверждал, что католическая государственная церковь, как правило, несовместима с демократией, так как «католицизм пытается контролировать столь многие стороны жизни, поощрять такую фиксацию статуса и подчинения авторитету, оставаться настолько независимым от мирской власти, что неизменно сталкивается с либерализмом, индивидуализмом, свободой, мобильностью и суверенитетом демократической нации» (Davis K. Political ambivalence in Latin America // J. of legal a. political sociology. - N.Y., 1942. - Vol. 1, N 1-2. - P. 127-150; цит. по: The evolution of Latin American government / Ed. by A.N. Christensen. -N.Y.: Holt, 1951. - P. 240).

социализм. Иначе говоря, там, где рабочим отказывали в экономических и политических правах гражданства, их борьба за перераспределение дохода и статуса накладывалась на революционную идеологию. Там, где экономическая и статусная борьба развивалась вне этого контекста, идеологией, с которой она соединялась, обычно была идеология постепенных реформ. В Германии Гогенцол-лернов, например, рабочим было отказано в свободном и равном избирательном праве в Пруссии - и так было до революции 1918 г. Этот отказ в «гражданстве» способствовал сохранению в тех частях Германии, где не существовало равного избирательного права, революционного марксизма. В южной Германии, где в конце XIX в. были предоставлены полные права гражданства, преобладал реформистский, демократический, нереволюционный социализм. Удержание революционных догм в немалой части Социал-демократической партии помогло ультралевым получить голос в партийном руководстве, позволило коммунистам обрести силу после поражения в войне и - что исторически, пожалуй, еще важнее -способствовало тому, что значительные сектора немецких средних классов оказались напуганы. Они опасались, что победа социалистов будет реально означать конец всех их привилегий и статуса.

Во Франции рабочие завоевали право голоса, но до окончания Второй мировой войны им было отказано в базовых экономических правах. Основные группы работодателей отказывали в легитимности французским профсоюзам и пытались ослабить или разрушить их после каждой их новой победы. Нестабильность французских профсоюзов и их постоянная нужда в сохранении воинственности рабочих, продиктованная задачами выживания, сделали рабочих доступными для более революционных и экстремистских политических групп. Доминирование коммунистов во французском рабочем движении может быть во многом объяснено тактикой французских деловых классов.

Представленные выше примеры не объясняют того, почему разные страны по-разному справлялись с основными национальными расколами. Их должно быть, однако, достаточно для иллюстрации важности гипотезы, связывающей предпосылки стабильного демократического правительства с основой общественного разнообразия. Там, где несколько исторических расколов смешиваются и создают основу для мировоззренческой политики, демократия бу-

дет нестабильной и слабой, ибо такие политические взгляды по определению не включают понятие толерантности.

Мировоззренческая политика ослабляла возможности стабильной демократии, так как партии, характеризующиеся тотальными идеологиями, часто пытались создать то, что Зигмунд Нойман назвал «интегрированной» средой, т.е. средой, в которой жизни их членов максимально замыкаются в деятельностях, связанных с идеологией. Эти деятельности основываются на допущении, что их сторонников важно изолировать от контакта с «ложью», выражаемой неверующими. Нойман предложил провести основополагающее аналитическое различие между представительскими (репре-зентационными) партиями, которые укрепляют демократию, и интеграционными, которые ее ослабляют1. Типичными примерами первых являются большинство партий англоязычных демократий и Скандинавии и большинство нерелигиозных центристских и консервативных партий. Функцией партии они считают прежде всего привлечение голосов во время выборов. Интеграционные партии, в свою очередь, озабочены тем, чтобы привести мир в соответствие со своей базовой философией, или Weltanschauung. Они не видят себя конкурентами в игре взаимных уступок в политике давлений, в которой все стороны принимают правила игры. Скорее они видят политическую или религиозную борьбу как состязание между божественной или исторической истиной, с одной стороны, и фундаментальной ошибкой - с другой. При таком понимании мира возникает необходимость пресекать соприкосновение их сторонников с перекрестными давлениями, идущими от подрывающего их веру контакта с ложью.

Двумя основными нетоталитарными группировками, которые приняли такую процедуру, были католики и социалисты. В общем и целом почти во всей Европе до 1939 г. католики и социалисты

1 См.: Neumann S. Die deutschen Parteien: Wesen und Wandel nach dem Kriege.-2. Aufl. - B.: Junker & Dünnhaupt, 1932, - где представлено различение интеграционных и представительских партий. Впоследствии Нойман провел различие между партиями «демократической интеграции» (католическими и социал-демократическими) и партиями «тотальной интеграции» (фашистскими и коммунистическими). См.: Neumann S. Toward a comparative study of political parties // Modern political parties / Ed. by S. Neumann. - Chicago (IL): Univ. of Chicago press, 1956. - P. 403-405.

пытались увеличить объемы внутрирелигиозных и внутриклассовых коммуникаций, создавая сеть социальных и экономических организаций, связанных с религией и партией, в пределах которой их последователи могли проживать всю свою жизнь. Австрия дает, вероятно, лучший пример ситуации, в которой две группы - социал-католики и социал-демократы - разошлись друг с другом по всем трем историческим проблемам и разделили страну на два враждебных лагеря, жизнедеятельность которых замыкалась в значительной мере в организациях, привязанных к соответствующей религии или партии1.

Тоталитарные организации - фашистские и коммунистические - всеми способами распространяли интеграционистский характер политической жизни. Они превосходили все другие группы в определении мира в терминах борьбы и в видении разлагающего влияния либо иудаизма, либо капитализма как фактора, требующего обособления истинно верующих.

Попытки демократических интеграционных партий изолировать свою социальную базу от сторонних давлений явно деструктивны для требований стабильной демократии, в которой есть переходы от одних выборов к другим и в которой разногласия между партиями имеют возможность со временем разрешаться. Изоляция может усиливать лояльность партии или церкви, но может также служить помехой для привлечения партией новых социальных слоев. Австрийская ситуация иллюстрирует также сбой в электоральном процессе, возникающий тогда, когда подавляющая часть электората втянута в интеграционные партии. Необходимые правила демократической политики допускают, что переход в обе стороны -в партию и из партии - возможен и принимается как должный. Партии, надеющиеся завоевать большинство с помощью демократических методов, должны в конце концов отказываться от инте-грационистских тенденций. Единственным оправданием обособления от остальной культуры является сильная приверженность идее о том, что партия обладает единственной истиной и что есть ряд основополагающих проблем, которые должны быть разрешены че-

1 См.: Gulick C. Austria from Hapsburg to Hitler. - Berkeley: Univ. of California press, 1948. Об их формуле примирения этого антагонизма после Второй мировой войны см.: Secher H.P. Coalition government: The case of the Second Austrian Republic // American political science rev. - Cambridge, 1958. - Vol. 52, N 3. - P. 791.

рез триумф исторической истины. Когда рабочий класс в разных странах обрел полное гражданство в политической и экономической сферах, социалистические партии Европы отбросили свой ин-теграционистский акцент. Единственные нетоталитарные партии, которые могут поддерживать и поддерживают подобный политический курс, - это религиозные партии, такие как католические партии или кальвинистская Антиреволюционная партия Голландии. Естественно, католическая церковь и голландская кальвинистская церковь не «демократичны» в сфере религии. Они настаивают на том, что есть одна-единственная истина, так же, как это делают в политике коммунисты и фашисты. Католики могут принимать исходные посылки политической демократии, но никогда не могут принять посылки веротерпимости. И там, где конфликт между религией и неверием видится католикам или иным верующим единственной истинной церкви как очевидный, демократический процесс сталкивается с реальной дилеммой. Многие политические проблемы, которые в одних странах легко решаются путем компромисса, в других странах усугубляются религиозной проблемой и не могут быть разрешены.

Острые формы раскола, порождаемые накоплением нерешенных проблем, создающим мировоззренческую политику, поддерживаются систематической сегрегацией разных слоев населения в организованных политических или религиозных анклавах. При этом надобно заметить, что и, наоборот, везде, где социальная структура как бы естественно «изолирует» индивидов или группы с одинаковыми политическими диспозиционными характеристиками от соприкосновения с иными воззрениями, те, кто таким образом изолируется, обычно поддерживают политических экстремистов.

Например, постоянно отмечалось, что рабочие в так называемых «обособленных» отраслях - шахтеры, моряки, рыбаки, лесорубы, пастухи и портовые рабочие - обычно оказывают всестороннюю поддержку левым тенденциям. В таких районах подавляющее большинство обычно голосует за коммунистов или социалистов, и иногда доходит до того, что мы находим в этих районах по сути «однопартийную» систему. Изоляция создается тем, что требования, связанные с трудовой деятельностью, заставляют рабочих в этих отраслях жить в сообществах, населенных преимущественно людьми того же рода занятий. И эта изоляция,

видимо, ослабляет для таких рабочих давления, побуждающие быть терпимыми к другим точкам зрения и давать в своем кругу приют иным тенденциям мысли; она делает их восприимчивыми к экстремистским версиям доктрины, которой придерживаются другие, менее изолированные члены их класса. Следует ожидать, что слои, наименее «космополитические» (т.е. наиболее изолированные) в своих политических диспозициях, будут слоями, наиболее склонными к принятию экстремизма. Политическая нетерпимость фермерских групп во времена кризиса может служить еще одной иллюстрацией этого паттерна, поскольку у фермеров, как и у рабочих в изолированных отраслях, политическая среда обычно более однородна, чем у людей наиболее городских родов занятий1.

Эти выводы подтверждаются далее исследованиями индивидуального поведения на выборах, показывающими, что индивиды, подверженные перекрестным давлениям - принадлежащие к группам с разнонаправленными предрасположенностями, имеющие друзей, поддерживающих разные партии, регулярно встречающиеся с пропагандой разных тенденций, - менее склонны к жестким политическим приверженностям2.

1 Эта тенденция очевидным образом варьирует в зависимости от городских сообществ, типа сельской стратификации и т.д. Обсуждение роли профессиональной гомогенности и политической коммуникации между фермерами см.: Lip-set S.M. Agrarian socialism. - Berkeley: Univ. of California press, 1950. - Ch. 10: Social structure and political activity. Данные о недемократических склонностях сельских населений см.: Stouffer S. Communism, conformity and civil liberties. -N.Y.: Doubleday, 1955. - P. 138-139. В отчете Национального института изучения общественного мнения Японии (A survey concerning the protection of civil liberties. -Tokyo: National public opinion institute of Japan, 1951. - Report N 26) сообщается о том, что фермеры до сих пор были профессиональной группой, наименее озабоченной гражданскими свободами. Карл Фридрих, объясняя силу национализма и нацизма в среде немецких фермеров, предполагает схожие факторы: что «сельское население более гомогенно, что оно содержит меньшее число аутсайдеров и чужаков, что у него гораздо меньше контактов с зарубежными странами и народами и, наконец, что его мобильность гораздо более ограниченна» (Friedrich C. The agricultural basis of emotional nationalism // Public opinion quart. - N.Y., 1937. - Vol. 1, N 1. - P. 50-51).

2 Наверное, первое общее утверждение о последствиях «перекрестных давлений» для индивидуального и группового поведения можно найти у Георга Зим-меля. См.: Simmel G. Conflict and the web of group affiliations. - Glencoe (IL): Free press, 1956. - P. 126-195. Интересным примером разрыва преемственности в соци-

Множественные политически несогласованные групповые аффилиации и лояльности - это стимулы, помогающие снизить эмоциональный накал и агрессивность, заключенные в политическом выборе. Так, в сегодняшней Германии рабочий-католик, движимый в двух направлениях, будет скорее всего голосовать за христианских демократов, но он гораздо терпимее относится к социал-демократам, чем типичный католик из среднего класса1. Там, где человек принадлежит к множеству разных групп, предрасполагающих его к одному и тому же политическому выбору, он оказывается в ситуации изолированного рабочего и гораздо менее склонен проявлять толерантность к мнениям оппозиции или равнодушно смотреть на возможность ее прихода к власти.

Имеющиеся данные говорят о том, что шансы стабильной демократии повышаются в той степени, в какой социальные страты, группы и индивиды имеют множество пересекающихся политически значимых аффилиаций. В той степени, в какой значительная часть населения разрывается между конфликтующими силами, группы и индивиды оказываются заинтересованы в снижении остроты политического конфликта2. Как отмечали Роберт Даль и Тол-котт Парсонс, такие группы и индивиды заинтересованы также в защите прав политических меньшинств3.

альных исследованиях служит то, что понятие перекрестных давлений использовалось еще Зиммелем, но позже пришлось открыть его заново в исследованиях голосования. Детальное обсуждение воздействия принадлежности ко многим группам на политический процесс вообще см.: Truman D. The governmental process. - N.Y.: Knopf, 1951.

1 См.: Linz J. The social basis of German politics: PhD thesis. - N.Y.: Columbia univ. press, 1958.

2 См.: Berelson B., Lazarsfeld P.A., McPhee W. Voting. - Chicago (IL): Univ. of Chicago press, 1954, - где отстаивается полезность перекрестного давления как объяснительного понятия. См. также: Handbook of social psychology / Lipset S.M., Linz J., Lazarsfeld P.A., Barton A. Psychology of voting. - Cambridge: Addison-Wesley, 1954. - Vol. 2, где предпринята попытка уточнить последствия разных групповых членств для поведения избирателей и дан обзор литературы.

3 Как пишет Даль, «если большинство индивидов в обществе идентифицируются больше чем с одной группой, то присутствует некоторая позитивная вероятность того, что любое большинство будет содержать индивидов, идентифицирующихся в связи с какой-то целью с находящимся под угрозой меньшинством. Члены такого меньшинства, твердо отдающие предпочтение своей альтернативе, будут давать знать о своих чувствах тем членам предполагаемого большинства,

Стабильная демократия требует относительно умеренного напряжения среди противоборствующих политических сил. А политической умеренности благоприятствует способность системы решать ключевые вопросы, вносящие раскол, прежде чем возникнут новые. В той степени, в какой разобщениям на почве религии, гражданства и «коллективного торга» открыта возможность накапливаться и усиливать друг друга как стимуляторам партийной вражды, система ослабляется. Чем больше усиливаются и связываются друг с другом источники раскола, тем меньше вероятность политической толерантности. Так же обстоит дело и на уровне группового и индивидуального поведения: чем больше изоляция от гетерогенных политических стимулов, тем больше эти фоновые факторы «группируются» в одном направлении и тем выше шансы на то, что группа или индивид будет придерживаться экстремистских воззрений. Эти две связи - одна на уровне партийных проблем, а другая на уровне партийной поддержки - объединяются тем, что партии, отражающие накопленные нерешенные проблемы, будут стремиться изолировать своих приверженцев от противостоящих им стимулов, чтобы не допустить контакта с «заблуждениями», а изолированные индивиды и группы будут усиливать тенденции нетерпимости в политической партийной системе. Условиями, максимизирующими в электорате политический космополитизм, служат рост урбанизации, образования, средств коммуникации и рост благосостояния. Большинство очевидных изолированных занятий - добыча угля, рубка леса, сельское хозяйство - относятся к числу «первичных» занятий, т.е. занятий, относительная доля кото-

которые тоже на некотором психологическом уровне идентифицируются с меньшинством. Некоторые из этих сочувствующих могут лишить своей поддержки альтернативу большинства, и большинство рассыплется». См.: Dahl R.A. A preface to democratic theory. - Chicago (IL): Univ. of Chicago press, 1956. - P. 104-105. Пар-сонс предполагает, что «чрезмерное усиление импликаций политического различия приводит в действие солидарности среди приверженцев двух партий, существующие на других, неполитических основаниях, и большинство начинает защищать меньшинства одного с ними рода, политически от него отличные». См. очерк Парсонса «Голосование и равновесие американской политической системы»: Parsons T. Voting and the equilibrium of the American political system // American voting behavior / Ed by E. Burdick, A. Brodbeck. - Glencoe (IL): Free press, 1959. -P. 80-120.

рых в совокупной рабочей силе резко падает в процессе экономического развития1.

Итак, мы снова видим, что факторы, вовлеченные в модернизацию, или экономическое развитие, тесно связаны с факторами, задействованными в исторической институционализации ценностей легитимности и терпимости. При этом всегда надо помнить, что корреляции - это лишь утверждения об относительных степенях соответствия и что еще одно условие политического действия состоит в том, что корреляция никогда не должна быть чрезмерно жесткой и лишать людей чувства того, что они могут изменить течение дел своими действиями. И еще этот факт низкой корреляции означает, что для аналитических целей важно сохранять разграничения между переменными, даже если они взаимно коррелируют. Например, предложенный здесь анализ раскола приводит к конкретным положениям о том, каким образом разные электоральные и конституционные упорядочения могут влиять на шансы демократии. Эти обобщения будут представлены в следующем разделе.

IV. Системы правления и демократия

iНе можете найти то, что вам нужно? Попробуйте сервис подбора литературы.

Из гипотезы о том, что пересекающиеся основания размежеваний лучше для жизнеспособности демократии, следует, что двухпартийные системы лучше многопартийных, что электоральные системы, заключающие в себе выборы должностных лиц на территориальной основе, предпочтительнее систем пропорционального представительства и что федерализм лучше унитарного государства. Оценивая эти положения, важно еще раз отметить, что при их выдвижении принимается, что все другие факторы остаются постоянными. Конечно, стабильные демократии совместимы с многопартийной системой, пропорциональным представительством и унитарным государством. И еще я бы утверждал, что такие вариации в системах правления, будучи значимыми, гораздо менее важны, чем вариации, вытекающие из тех базовых различий в социальной структуре, о которых шла речь в предыдущих разделах статьи.

Довод в пользу двухпартийной системы базируется на ряде допущений: что в сложном обществе такие партии неизбежно

1 Clark C. The conditions of economic progress. - N.Y.: Macmillan, 1940.

должны быть широкими коалициями; что они не могут стремиться служить интересам только одной важной группы; что они не могут быть интеграционными партиями; что при образовании предвыборных коалиций они с необходимостью противостоят опоре на тех, кто наиболее им привержен, и наоборот - должны стремиться завоевать поддержку в группах, которые солидаризуются преимущественно с противостоящей партией. Так, британские консерваторы и американские республиканцы не должны предпринимать действий, всерьез настраивающих против них работников физического труда, поскольку значительную часть голосов они получают от них. Демократы и лейбористы сталкиваются со схожей проблемой во взаимодействии со средними слоями. Партии, никогда не ориентирующиеся на получение большинства, стремятся максимизировать свою электоральную поддержку в ограниченной базе. Так, ориентированная на крестьян партия будет акцентировать осознание интересов крестьянской группы, а партия, взывающая прежде всего к малому бизнесу, будет делать то же в отношении своей группы. Выборы, вместо того чтобы быть важным событием, в связи с которым партии стремятся обзавестись как можно более широкой базой поддержки и тем самым подвести дивергентные группы к осознанию их общих интересов, становятся событиями, в которых партии подчеркивают размежевания, отделяющие их принципиальных сторонников от прочих группирований.

Положение о том, что пропорциональное представительство скорее ослабляет, чем укрепляет демократию, базируется на анализе различий между многопартийной и мажоритарной ситуациями. Если и в самом деле, как выше предполагалось, «многопартийность» служит обострению различий и подрывает консенсус, то любая избирательная система, повышающая шанс для больше чем небольшого числа партий, плохо служит демократии.

Кроме того, как отмечал Георг Зиммель, система выбора в парламент членов, представляющих территориальные образования, в отличие от систем, поощряющих прямое представительство групп (например, пропорциональное представительство), предпочтительна, так как представительство территорий способствует стабилизации политической системы, принуждая группы интересов к тому, чтобы они достигали своих целей только в электоральных

рамках, заключающих в себе некоторую заботу о множественных интересах и потребность в компромиссе1.

Федерализм служит укреплению демократии, повышая вероятность наличия множественных источников раскола. Он добавляет региональные интересы и ценности к другим - таким как класс, религия и этничность, - пронизывающим всю социальную структуру.

Основное исключение из этого обобщения проявляется тогда, когда федерализм делит страну по линиям базового раскола, например по границам между разными этническими, религиозными или языковыми ареалами. Так, например, в Индии и Канаде федерализм может акцентировать и усугублять расколы. Желательно, чтобы раскол был внутри языковых или религиозных групп, а не между ними. При этом если таких разделений не существует, федерализм, видимо, хорошо служит демократии. Наряду с созданием еще одного источника размежевания он выполняет также различные функции, которые, как отмечал Токвиль, роднят его с сильными добровольными ассоциациями. Он служит источником сопротивления централизации власти и источником подготовки новых политических лидеров; кроме того, он дает «проигравшей» партии долю в системе в целом, поскольку «проигравшие» общенациональные партии обычно продолжают контролировать некоторые единицы системы.

Повторю еще раз, что я вовсе не имею в виду, будто эти аспекты политической структуры сами по себе являются ключевыми условиями демократических систем. Если основополагающие социальные условия способствуют демократии (что, видимо, верно для Швеции), то сочетание многопартийности, пропорционального представительства и унитарного государства не ослабляет ее всерь-

1 Simmel G. Conflict and the web of group affiliations. - Glencoe (IL): Free press, 1956. - P. 191-194. Толкотт Парсонс выступил недавно с таким же тезисом, указав на то, что одним из механизмов предотвращения «прогрессивно углубляющегося раскола в электорате» является «такое сплетение голосования со сложной социетальной структурой солидарности, при котором между политической поляризацией и другими основаниями дифференциации была бы корреляция, но не было точного соответствия» (Parsons T. Voting and the equilibrium of the American political system // American voting behavior / Ed by E. Burdick, A. Brodbeck. -Glencoe (IL): Free press, 1959. - P. 80-120.)

ез. Самое большее, оно позволяет не несущим ответственности меньшинствам заполучить плацдарм в парламенте. С другой стороны, там, где низкий уровень эффективности и легитимности способствует ослаблению основ демократии, как произошло в Веймарской Германии или во Франции, там конституционные факторы, поощряющие многопартийность, служат уменьшению шансов на то, что система выживет.

V. Проблемы сегодняшней демократии

Для стабильных западных демократий середины XX в. характерен паттерн «постполитической» фазы: между демократическими левыми и правыми относительно мало разницы, социалисты умеренны, а консерваторы принимают государство благосостояния. В значительной мере это отражает тот факт, что в этих странах рабочие победили в своей борьбе за гражданство и за доступ к политике, т.е. за право наравне с другими принимать участие во всех государственных решениях1.

В борьбе за гражданство было два аспекта: политический (доступ к власти через избирательное право) и экономический (ин-ституционализация права профсоюзов участвовать в принятии решений, влияющих на трудовое вознаграждение и условия труда). В настоящее время представители низших страт являются частью правящих классов, членами клуба. В самых богатых стабильных демократиях политическое противостояние пришло в упадок, так как основная политическая проблема промышленной революции -включение рабочих в легитимное «политическое тело» - была в них решена. Сегодня единственной ключевой внутренней проблемой является коллективный торг вокруг различий в распределении

1 Т.Х. Маршалл проанализировал постепенный процесс инкорпорации рабочего класса в «политическое тело» в XIX в. и увидел в этом процессе достижение «основополагающего человеческого равенства, связанного с полным членством в сообществе, не являющегося несовместимым с надстройкой экономического неравенства». См. его небольшую, но блестящую книгу: Marshall T.H. Citizenship and social class. - Cambridge: Cambridge univ. press, 1950. - P. 77. Но даже если универсальное гражданство и открывает путь к оспариванию оставшихся социальных неравенств, оно одновременно дает основу и для веры в то, что процесс социального изменения в направлении равенства будет оставаться в границах конфликта, допустимого для демократической системы.

совокупного продукта в структурных рамках кейнсианского государства благосостояния; а такие проблемы не требуют экстремизма ни от одной из сторон и не способствуют ему.

В большей части латинской и Восточной Европы борьба за интеграцию рабочего класса в «политическое тело» не достигала успешного завершения раньше, чем коммунисты выходили на сцену и захватывали лидерство в рабочей среде. Этот факт кардинально менял политическую игру, поскольку коммунисты принципиально не могли быть абсорбированы в систему тем способом, которым в нее были абсорбированы социалисты. Демократическое общество, возможно, вообще не может дать право доступа коммунистически настроенным рабочим, их партиям и профсоюзам. Восприятие коммунистами самих себя и особенно их связи с Советским Союзом ведут их к принятию самоподтверждающейся гипотезы: их самоопределение не позволяет им получить доступ, а это в свою очередь усиливает то чувство отчуждения от системы (непринятия другими слоями), которым обладают рабочие в нациях с крупными коммунистическими партиями. Более консервативные слои при этом укрепляются во мнении о том, что предоставление больших прав рабочим или их представителям несет угрозу всему, что есть в жизни хорошего. Итак, присутствие коммунистов не позволяет с легким сердцем предсказывать, что экономическое развитие в этих европейских странах будет стабилизировать демократию.

В новоиспеченных независимых нациях Азии ситуация несколько иная. В Европе на заре современной политики перед рабочими стояла проблема завоевания гражданства, т. е. права принимать участие в политической игре, у господствовавших аристократических и деловых слоев, контролировавших политику. В Азии долгое присутствие колониальных властей отождествило консерватизм как идеологию и более состоятельные классы с подчинением колониализму, а левые идеологии, обычно в марксистской разновидности, стали господствующими, отождествившись с национализмом. Профсоюзы и рабочие партии в Азии с самого зарождения демократической системы были частью политического процесса. Такая ситуация, видимо, могла бы означать стабильную демократию, если бы не тот факт, что эти права низших слоев опережают развитие стабильной экономики с большим средним классом и индустриального общества.

Вся система держится на верхушке. Левые в европейских стабильных демократиях постепенно вырастали в борьбе за большую демократию, давая выражение недовольствам, связанным с ранней индустриализацией, а правые сохраняли поддержку со стороны традиционных элементов общества, пока в конце концов система не вошла в состояние непринужденного равновесия между преображенными левыми и правыми. В Азии левые приходят к власти в период популяционного взрыва и ранней индустриализации и должны брать ответственность за все вытекающие невзгоды. Как и в сравнительно бедных ареалах Европы, в большинстве азиатских государств коммунисты совершенно безответственным образом спекулируют на этих недовольствах и являются в настоящее время важной партией, обычно второй по величине.

Если принять во внимание наличие нищенствующих масс, низкие уровни образования, классовую структуру в виде вытянутой пирамиды и «преждевременный» триумф демократических левых, то прогноз сохранения политической демократии в Азии и Африке будет безрадостным. Нации, имеющие в этом отношении наилучшие перспективы - Израиль, Япония, Ливан, Филиппины и Турция, -обычно схожи с Европой в одном или более основных факторах, как то: высокий образовательный уровень (все кроме Турции), существенный и продолжающий расти средний класс и удержание политической легитимности не-левацкими группами. Другие зарождающиеся национальные государства Азии и Африки привержены некоторому темпу и паттерну экономического развития и национальной независимости, неважно в какой политической форме, глубже, чем паттерну партийной политики и свободных выборов, отличающему нашу модель демократии. Представляется вероятным, что в странах, избежавших коммунистической или военной диктатуры, политическое развитие будет следовать паттерну, развившемуся в таких странах, как Гана, Тунис или Мексика, где образованное меньшинство способно оседлать массовое движение, выступая с левацкими лозунгами ради эффективного контроля над ним, и сохраняет выборы как жест ориентации на высшие демократические цели и средство оценки общественного мнения, а не как действенный инструмент легитимной смены одних правящих пар-

тий другими на высших постах1. В условиях давления, побуждающего к быстрой индустриализации и немедленному решению хронических проблем нищеты и голода политическими средствами, вряд ли многие из новых правительств Азии и Африки будут характеризоваться открытой партийной системой, представляющей существенно разные классовые позиции и ценности2.

Латинская Америка, столь же экономически недоразвитая, как и Азия, в политическом отношении, однако, больше похожа на Европу начала XIX в., чем на сегодняшнюю Азию. Большинство латиноамериканских стран стали независимыми государствами до подъема индустриализма и марксистских идеологий и содержат твердыни традиционного консерватизма. Сельские местности в них часто аполитичны или традиционны, а левацкие движения находят поддержку прежде всего в промышленном пролетариате. Например, латиноамериканские коммунисты выбрали европейский марксистский путь организации городских рабочих, а не «янаньский

1 Обсуждение развития политических паттернов в Гане см. в работе: Ap-ter D. The Gold Coast in transition. - Princeton (NJ): Princeton univ. press, 1955. Интересный краткий анализ мексиканской «однопартийной» системы см.: Padgett L.V. Mexico's one-party system, a re-evaluation // American political science rev. - Cambridge, 1957. - Vol. 51, N 4. - P. 995-1008.

2 Пока эта статья готовилась к публикации, в нескольких бедных и неграмотных странах разразились политические кризисы, еще раз подчеркнув нестабильность демократических правительств в недоразвитых ареалах. 7 октября 1958 г. было мирно низвергнуто правительство Пакистана, и новый самозваный президент заявил, что «в нынешних условиях демократия западного типа не может здесь функционировать. У нас грамотны всего 16%. В Америке - 98%» (Associated Press release. - N.Y., 1958. - Oct. 9). Далее новое правительство распустило парламент и все политические партии. Почти одновременно схожие кризисы разразились в Тунисе, Гане и даже в Бирме, правительство которой считалось после Второй мировой войны, при премьере У Ну, одним из самых стабильных в Юго-Восточной Азии. Гвинея начала жизнь независимого государства с однопартийной системой.

Возможно, открытое возникновение полудиктатур без сколько-нибудь значимого демократического «фронта» отражает ослабление демократических символов в этих ареалах под влиянием советской идеологии, приравнивающей «демократию» к быстрому, эффективному осуществлению «воли народа» образованной элитой, а не к особым политическим формам и методам.

путь» Мао, опирающийся на крестьян1. Если Латинской Америке будет дана возможность развиваться своим путем и она сможет повысить свою производительность и нарастить средние классы, то высоки шансы на то, что многие латиноамериканские страны пойдут в европейском направлении. Недавние изменения, в том числе свержение нескольких диктатур, в значительной мере отражают влияние роста среднего класса, благосостояния и уровня образованности. Есть вместе с тем и возможность того, что эти страны пойдут скорее во французском и итальянском направлении, чем в североевропейском, что ведущие позиции среди рабочих захватят коммунисты и что средний класс будет отчужден от демократии.

Анализ социальных предпосылок демократии, содержащийся в этой статье, был нацелен на выявление некоторых, хотя, конечно, далеко не всех структурных условий, связанных с этой политической системой. Мы имели лишь очень ограниченную возможность попытаться проверить выдвигаемые гипотезы. Эти предварительные попытки применения научного метода для сравнения политических систем могут на данном этапе рассматриваться как сугубо иллюстративные, ибо мы мало что можем сказать о действительных вариациях в национальных социальных структурах. Требуется значительно больше исследований, уточняющих по многим параметрам границы разных обществ, прежде чем можно будет провести надежный сравнительный анализ того типа, который был здесь намечен. Хотя задача эта, несомненно, связана с огромными трудностями, только с помощью таких методов мы сможем выйти за рамки принятых полуграмотных методов подбора иллюстративных примеров в поддержку правдоподобных интерпретаций.

Между тем имеющиеся данные достаточно согласуются друг с другом, чтобы поддержать вывод о том, что более систематическая и современная версия гипотезы Аристотеля о связи политических форм с социальной структурой валидна. К сожалению, как уже отмечалось выше, этот вывод не оправдывает оптимистических надежд либералов на то, что рост благосостояния, размеров среднего класса, образования и других связанных с ними факторов будет с необходимостью означать распространение демократии и

1 Alexander R.J. Communism in Latin America. - New Brunswick (NJ): Rutgers univ. press, 1957.

ее стабилизацию. Как отмечал Макс Вебер, обсуждая шансы на демократию в России начала ХХ в., «распространение западной культуры и капиталистической экономики ipso facto не гарантировало, что Россия приобретет также свободы, которые сопровождали их появление в европейской истории... Европейская свобода родилась в уникальных, возможно неповторимых обстоятельствах в то время, когда материальные и духовные условия были исключительно благоприятны для нее»1.

Предположение о том, что особая констелляция факторов, породившая в XIX в. западную демократию, возможно, уникальна, не должно быть поводом для неуместного пессимизма. Политическая демократия существует и до сих пор существовала во множестве разных обстоятельств, пусть даже чаще всего она и поддерживается ограниченным кластером условий. Если полнее понять различные условия, при которых она существовала, то это может сделать возможным развитие демократии и в других местах. Демократии не достичь одними актами воли; но воли людей могут через действие формировать институты и события в направлениях, снижающих или повышающих шансы демократии на развитие и выживание. Помочь людям в развитии демократии было в какой-то мере целью Токвиля при изучении того, как работает демократия в Америке, и это остается, возможно, важнейшей существенной интеллектуальной задачей, которую по сей день могут ставить перед собой исследователи политики.

Методологическое приложение

Принятый в этой статье подход (как уже отмечалось) неявным образом отличается от других, применявшихся при попытках работать с социальными феноменами на уровне всего общества, и, наверное, будет полезно прояснить некоторые из методологических постулатов, лежащих в его основе.

Для рассмотрения сложных характеристик социальной системы, таких как демократия, степень бюрократизации, тип стратификационной системы, обычно применяли редукционистский или «идеально-типический» подходы. Первый упускает из виду воз-

1 Pipes R. Max Weber and Russia // World politics. - Cambridge, 1955. -Vol. 7, N 3. - P. 383.

можность рассмотрения этих характеристик как атрибутов системы и утверждает, что сумму и содержание социологических категорий составляют качества индивидуальных действий. Для этой школы мысли распространенность демократических установок, бюрократического поведения или количество и типы рангов престижа или власти составляют существенный смысл атрибутов демократии, бюрократии или класса.

«Идеально-типический» подход отталкивается от схожих допущений, но приходит к противоположному выводу. Схожее допущение состоит в том, что общества - это сложный разряд феноменов, проявляющих такую степень внутренней противоречивости, что обобщения о них как о целостностях с необходимостью должны образовывать сконструированную репрезентацию отобранных элементов, проистекающую из особых интересов и перспектив ученого. Противоположный вывод гласит, что у такого рода абстракций, как «демократия» или «бюрократия», нет необходимой связи с действительно существующими состояниями или качествами сложных социальных систем, а они содержат в себе наборы атрибутов, логически взаимосвязанных, но не характеризующих в своей совокупности ни одно существующее общество1. Примером такой абстракции является веберовское понятие «бюрократии», заключающее в себе некоторое множество «должностей», не находящихся во «владении» занимающих их лиц, непрерывно ведущуюся документацию, функционально специфицированные обязанности и т. д. Другой пример - это общее определение демократии в политической науке, которое постулирует индивидуальные политические решения, основывающиеся на рациональном знании собственных целей и фактической политической ситуации.

Критика подобных категорий, или идеальных типов, исключительно на основе того, что они не соотносятся с реальностью, неуместна, поскольку их задача не в том, чтобы описать реальность, а в том, чтобы обеспечить основу для сравнения разных аспектов реальности в их отклонениях от логически консистентного чистого случая. Часто этот подход довольно плодотворен, и здесь у

1 См. очерк М. Вебера «"Объективность" социально-научного и социально-политического познания»: Weber M. «Objectivity» in social science and social policy // Weber M. Methodology of the social sciences. - Glencoe (IL): Free press, 1949. -P. 72-93.

нас нет намерения поставить на его место другой методологический подход; мы собираемся всего лишь представить еще один возможный способ концептуализации сложных характеристик социальных систем, вытекающий из многомерного анализа, впервые апробированного на совершенно другом уровне анализа Полом Ла-зарсфельдом и его коллегами1.

Отличие этого подхода обнаруживается в ответе на вопрос: можно ли рассматривать обобщенные теоретические категории как имеющие валидную связь с характеристиками целостных социальных систем? Представленные в этой статье статистические данные о демократии и связи между демократией, экономическим развитием и политической легитимностью говорят о том, что в целостных социальных системах существуют аспекты, которые могут быть установлены в теоретических терминах, сопоставлены со схожими аспектами других систем и в то же время выводимы из эмпирических данных, могущих быть проверенными (или оспоренными) другими исследователями. Это не означает, что не может быть ситуаций, противоречащих общей связи, или что на низших уровнях социальной организации не могут явно обнаруживаться совершенно другие характеристики. Например, такая страна, как США, может быть охарактеризована как «демократическая» на общенациональном уровне, даже если большинство вторичных организаций в стране могут не быть демократическими. На другом уровне церковь может характеризоваться как «небюрократическая» организация по сравнению с корпорацией, даже если важные сегменты церковной организации могут быть в такой же мере бюрократизированными, как и наиболее бюрократические части корпорации. На еще одном уровне может быть вполне законным в целях психологической оценки личности в целом рассмотреть некоего индивида как «ши-

1 Методологические пресуппозиции этого подхода на уровне многомерных корреляций и взаимодействий индивидуального поведения с разными социальными характеристиками были представлены в: Lazarsfeld P.F. Interpretation of statistical relations as a research operation // The language of social research / Ed. by P.F. Lazarsfeld, M. Rosenberg. - Glencoe (IL): Free press, 1955. - P. 115-125; Hyman H. Survey design and analysis. - Glencoe (IL): Free press, 1955. - Ch. 6, 7. См. также методологические приложения в: Lipset S.M. The political process in trade unions: A theoretical statement // Freedom and control in modern society / Ed. by M. Berger. - N.Y.: Van Nostrand, 1954. - P. 122-124.

зофреника», даже если в каких-то условиях он может не вести себя шизофренически. Суть в том, что при проведении сравнений на уровне обобщения, относящемся к функционированию целостной системы (будь то на уровне личности, группы, организации или общества), обобщения, применимые ко всему обществу, не отличаются по типу и степени валидности от обобщений, применимых к другим системам, и должны так же эмпирически проверяться. Увидеть это мешало отсутствие достаточного количества систематических сравнительных исследований нескольких обществ.

Условия Исходные возможные Дополнительные

следствия следствия

открытая классовая система

экономическое благосостояние

эгалитарная ценностная система

капиталистическая экономика

грамотность

высокая степень участия в добровольных организациях

открытая классовая система

эгалитарная ценностная система

политическая апатия

демократия

бюрократия

массовое общество" •4-

амотность

Этот подход также подчеркивает, что комплексные характеристики целостной системы имеют как многомерные причины, так и многомерные последствия, поскольку такие характеристики обладают некоторой степенью автономии внутри системы. Бюрокра-

тия и урбанизация, как и демократия, имеют в этом смысле много причин и следствий1.

С этой точки зрения, трудно было бы идентифицировать какой-то один фактор, решающим образом связанный с той или иной сложной социальной характеристикой или «причинно вызывающий» ее. Скорее все такие характеристики (и это методологическое допущение, призванное направлять исследование, а не содержательное суждение) считаются имеющими много причин и множество следствий. Этот момент можно прояснить с помощью рисунка, на котором представлены некоторые из возможных связей между демократией, исходными условиями, ассоциированными с ее появлением, и следствиями существования демократической системы.

Появление фактора по обе стороны от «демократии» означает, что он является предпосылкой демократии, а демократия, однажды установившись, поддерживает эту характеристику общества, например открытую классовую систему. С другой стороны, некоторые исходные последствия демократии, как, например, бюрократия, могут, в свою очередь, иметь следствием подрыв демократии, на что указывают обратные стрелки. Появление некоего фактора справа от «демократии» не означает, что демократия служит «причиной» его появления, а означает лишь, что демократия является предпосылкой, которая благоприятствует его развитию. Схожим образом гипотеза о том, что бюрократия есть одно из последствий демократии, не означает, что демократия является единственной ее причиной, а означает, скорее, что демократическая система имеет следствием поощрение развития некоторого типа бюрократии при прочих дополнительных условиях, которые должны быть установлены, если бюрократия попадает в фокус исследовательской про-

1 Этот подход отличен от попытки Вебера проследить истоки современного капитализма. Вебера интересовало установление того, что один предшествующий фактор, а именно определенная религиозная этика, имел решающее значение в синдроме экономических, политических и культурных условий, ведущих к развитию западного капитализма. Меня же интересует не установление каузальной необходимости какого-то одного фактора, а скорее синдром условий, чаще всего отличающих нации, которые можно эмпирически категоризировать как «более демократичные» или «менее демократичные», не закладывая в это определение каких-либо абсолютных качеств.

блемы. Этот рисунок не претендует на то, чтобы быть полной моделью общих социальных условий, связанных с возникновением демократии; его цель - придать наглядность методологическому тезису о многомерности связей в целостной социальной системе.

Итак, в многомерной системе можно сосредоточить внимание на одном элементе и устанавливать его условия и последствия, не предполагая, что мы пришли к завершенной теории необходимых и достаточных условий его появления. В этой статье нет попытки выдвинуть новую теорию демократии; мы всего лишь пытались формализовать и эмпирически проверить некоторое множество связей, предполагаемых традиционными теориями, на уровне целостных социальных систем.

Пер. с англ. В.Г. Николаева

2012.03.004. ЛИПСЕТ СМ. ЕЩЕ РАЗ О СОЦИАЛЬНЫХ ПРЕДПОСЫЛКАХ ДЕМОКРАТИИ.

LIPSET S.M. The social requisites of democracy revisited // American sociological rev. - Chicago (IL), 1994. - Vol. 59, N 1. - P. 1-22.

В статье, ставшей его президентским обращением к Американской социологической ассоциации 1993 г., Сеймур Мартин Липсет возвращается к теме своей ранней работы «Некоторые социальные предпосылки демократии»1. Спустя 35 лет после опубликования этой статьи ее автор предлагает свежие интерпретации выдвинутых им ранее положений с учетом нового исторического опыта и результатов новейших социальных исследований.

В свете «третьей волны»2 демократизации, начавшейся в середине 1970-х годов в Южной Европе и продолжившейся в 1980-е годы в Латинской Америке и Азии, а в конце 1980-х - начале 1990-х годов в Восточной Европе, СССР и Африке южнее Сахары, центральным предметом обсуждения становятся факторы и процессы, влияющие на перспективы институционализации демократии в этих регионах. При том, что к концу 1993 г. в 107 из 186 стран мира (что вдвое больше, чем в 1970 г.) существовали «свободные выборы и различные гарантии политических и индивидуальных прав»,

1 Lipset S.M. Some social requisites of democracy: Economic development and political legitimacy // American political science rev. - Cambridge, 1959. - Vol. 53, N 1. - P. 69-105.

2 TepMUH C. XaHiTHHiTOHa.

i Надоели баннеры? Вы всегда можете отключить рекламу.